You are on page 1of 252
from THE MATHEMATICS TEACHER INTER-II ee SC GEMS From THE MATHEMATICS TEACHER INTER -II Edited by Sri. V. Seshan THE ASSOCATION OF MATHEMATICS TEACHERS OF INDIA B-19, Vijay Avenue, 85/37, Venkatarangam Street, Triplicane, Chennai — 600 005. Tel: (044)-28441523 E-mail : amti@vsnl.com Web site: amtionline.com LA BETWEEN US Dear Reader, The demand for our books based on NMTC questions had been increasing steadily. Hence it is decided to bring all the problems and solutions of NMIC every five or ten years without disturbing the earlier editions. To satisfy the needs of various age groups they are being brought out in five different books - Primary, Sub-Junior, Junior, Inter ‘and RMO and INMO. The Present period covered by these books are from 2004 to 2012. In the case of RMO and INMO it is 2005 to 2013. We hope and trust the beneficiaries of our service will continue to encourage us to help aspirants of excellence in Mathematics Education. We are grateful to Smt. K._— Maheswari, Sri G. Gnanasundaram, Dr. M. Palanivasan and Sri V. Seshan respectively for having gone through the pages and edited these books in record time, With best wishes. Yours sincerely, ret (M. MAHADEVAN) General Secretary, AMTI CONTENTS 1. Year 2004 Screening Test . Final Test .. 2. Year 2005 Screening Test...... Final Test....... 3. Year 2006 Screening Test . Final Tes! 4. Year 2007 5. Year 2008 6. Year 2009 7. Year 2010 Screening Test Final Test 8. Year 2011 Screening Test Final Test 9. Year 2012 Screening Test... Final Test RAMANUJAN CONTEST SCREENING TEST - 2004 1. When 12003 +. 22003 4 32003 + ... + 290370 ja divided by 2004, then the remainder is (A) 0° (B)1 (G) 1002 (D)2008 Solution: When in is odd, a” + 8” is divisible by a +b. So, 1208 +. (2004—m)?%08 is divisible by 2004 for n= 1,2,--+ ,1001. ‘Also 1002? is 1002 x 1002 x 1002! = 2004 x 501 x 10027, Hence it is also divisible by'2004. «1, Given sum’= (1208 4 29932008) 4. (22008 + 29027003) 4.0.4 (100173 + 100303) +1002 is also divisible by 2004. Answer: (A) 2. Rach number from 1,2,3,-** ,100(decimal scale) is written in base 6 and their product is also written in base 6. Then the number of zeroes at the end of this product is (A) 24 (B) 48 (Cc) 18 (D)97 Solution: The problem is equivalent to finding m such that when 100! is divided by 67, the. quotient is not divisible by 6. This is equivalent to finding-the greatest mi such that 124 is not divisible by 3. : Now 81 = 34, 27 = 38, 54=2 38, 9=9?, 18 = 2x 3%, 36 = 2 x37, 45 = 5 x 9,63 = 7x 3, .72 = 2 x 3%, 99 = 2x 5x 34, 99 = 11 x 3? while remaining multiples of 3 liave only one 3 as factor. The total-number of multiples: of 3 in 05 co Problems and Solutions 1,2,+++ 100 being 33 the numbers having just one 3 as factor will be 33 — 11 = 22, Hence total power of 3 in the product 100! will be ; 224-4 (from 81)+2x3(from 27 and 54)+8x2(Grom 9, 18,-+~ 99) = 24446416 = 48, Hence the number of zeros at the end of 100! of decimal expressed in base 6 will be 48. Answer: (B) . Given that aya2,--- ,a2094 are distinct positive real numbers then 21+ 224... 4 gum. 4 sans jg (A) less than 2004 (B) less than 1 (C) greater than 2004 (D) equal to 1 Solution: Let eq = g242,m = 1,2,++- ,2003, Then 4.09 = Be at fm ay tp.t9-+ 9008 Using the property thet A.M> G.M, we get {21 +22+-+-+ 22003 + ==} —— SE 2004 > {x10 + 2003-5 1 “taney! = omtmte om + 22 5 as i. 21g 2g RD 5 ME apg a ag 22004 a Equality can hold only if each term on the left hand side is 1 Je., only if ¢) = aq = --- = azogy. But, we are given that the feta Stata a et positive real numbers. Hence at: Banos: 3 ata +p Stto8 ee 7008 > 2004, Answer: (C) Problems and Solutions 4, Two chords of acircle bisect each other. Then (A) The chords are of equal length but less than the length of the diameter. (B) The chords are of unequal length. (C) The chords should be perpendicular bisectors of each other. (D) The chords are any two diemeters. Solution: Let AB,CD be the chords intersecting at P such that AP = PB,OP =, PD: As PG.PD = PA.PB, we get PA? = PC? or PA = PC. This means YP AB = CD, i‘e., the chords are of equal length. ‘The perpendiculars to AB and CD at their midpoints pass through the centre of the circle. Unless P is the centre of the circle this cannct happen. Thus ‘AB and CD are diameters.of the circle, They need not be mutuelly perpendicular. Thus option (D) is the correct one. Answer: (D) 5. A man tosses a fair coin till he gets a head. ‘The probability that he gets head in at most n tosses is (Ads ®)* (1-3 (D)1-* Solution: If P(Z) denotes the probability of event E, then P(Head is got in at most 7 tosses) is equivalent to the sum of the probabilities for the n mutually exclusive cases of getting tails in the first r tosses and then head in the (r + 1)th toss, r=Q1,++,n-1. Problems and Solutions Thus the required probability nel - = yp (getting tails in the first r tosses and then head 70 in the (r + 1)th tosses) - > LiL $ 1 =e a> Da eo yh : Note that P(Head in a toss) = P (Tail in a toss) = } and the tosses in each case are independent. «, Required probability = jtpetethé a2-G@")_, 1 ~ G-f ( ) Answer: (D) 11. Kumar never lies except ou ‘Tuesdays. On Tuesdays, he alvays lies. On how many days of the week can he say “If I did not lie yesterday then I will lie tomorrow* (A)1 (B)2 ()3 (D) 4 Solution: If the statement is true Kumar could have spoken this truth only on Monday. If the statement is false, Kumar could have lied only on Tuesday. Thus his statement given in the problem could have been given only on two days, Answer: (B) 12, The range of function f(¢) =7.r Ag where mP, represents Permutations of ‘ra’ things taken ‘n’ at a time end where ris 4 non-negative integer is ‘ (A) 0,23,4,5} (8) (1,234) (GQ, 2} (D) {1,2,3) Problems and Solutions g Solution: For f(r) =y-1 Prs to be defined, 7—r > 17-3 orr <5, Also r—3 > 0 implies r > 3, Thus domain of the function f is {3,4,5} and range of f is {F(3),f(4), f(5)} i.e, {4Po, 8P,,2Pa} ive, {1,3, 2} Answer: (D) 18: The roots of 642° ~ 1442? + 92¢ — 15 = O-are in A.P. Then the difference between the lergest and the smallest is Aa (8) 4 5 @y Solution: As the roots.are in‘A.P., take them as a—d, o and o+d, Then ; Sum of the roots =3a = Yt = 3 or + §. Product. of.the roots=a(a?— d®) = 2B... Asa=3; f= §- R= gad. wae th. “The difference’ between the largest and the smaliést of the roots Ko+d = (eal = 2a] = 2] #3} = 1. Answer: (D) 14, In the’ following figure, if “AD=DC =z and BO=y then AB =? (A)a+y (B) yt Ss (©) 2y (D) 22 . Solution: From isosceles triangles ABC and ADC we Reve mZABC = mZAGB = 80° and mZDAC = mZDGA -= 40°, " Therefore, mZBAD = 60° and mZBCD = 120° Let AB =z. .Then AC’ = z. Using cosine formule, BD? can be got from AABD arid ABCD, ‘Eqiating these two’ values of BD?, we have Problems and Solutions 10 22422 2z¢cos60? = 2?+y?— 2zycos 120° i 2492 1 rer) 1 ie, 2+ tas = oy’ —2y, >) (F-y)-a(y+2) = Oor(z-y—a)(yt+2)=0 Asy+2¢0, 2=2+y, Answer: (A) 15. ABCD is a tetrahedron. The number of planes from which the distances to A, B,C, D are equal is (A)o (B) 5 (c)4 (D)3 Solution: Let ABCD be the tetrahedron and E and F be the midpoints of BD and DG respectively. Then every plane intersecting the plane BDC along EF will be at the same distance, say p, from B,D and C. Of these one plane alone will be at distance p from A also, Thus for every face of tetrahedron ABCD there is one plane satisfying the conditions of the problem. As there are four faces for the tetrahedron, there are four such planes. Answer:. (C) 16. The three last digits of 7°? are (A) 263 (B) 148 (C) 343 (D) 523 Solution: The last three digits are given by the number got when 79°99 is expressed congruent modulo 1000. Now 74 = 2401 = 401(mod 1000), 78 = 801(mod1000), 7"? = 201(mod1000), 7 = 601(mod1000) and 7° = 001(modi000). This implies that 7°" = 1(mod1000) for any integer n > 0. Let 79% = x(mod 1000). Then 7710000 = (72)500 — 7—(mod 1000). Problems and Solutions 11 ‘This implies that'7s = 1(mod'1000)as 7°° = 1(mod 1000) 143 x 7a" 143(motl 1000) or 10012 = 143(mod 1000). or 1000z + x = 143(mod 1000) ie,, ¢ =143(mod1000)' oo ~. Last three digits of 79° are 143. Answer: (B) 17. Each of.the faces of a cube is coloured by a different colour. How:many of.ttie colourings are distinct? (A) 6 (B) 30 - (C18 (D) 24 Solution; Let'the different colours he denoted by A, B, C;DE and F, If A and B are painted on adjacent faces, then the remaining four colours can be painted om the remaining: four faces in 4! = 24 ways. If A and.Biare painted:on two. opposite faces and considering C as painted on an adjacent face to them, the remaining three faces ¢an be painted in’3! +'6 ways, Thus the,total ‘ntimber of distinct’ colourings will be 24 4 6'= 90: Answer: (B) 16. "Which of:the follawing is @ continuous fubction { eatisfyitg 3f(2e-+-4)= f(a) + 52? (A) f(z) = ant 8 (BY f(z)=o+1 (C) fz) =2~$ ©) f@)s2t+4 Solution: Given $f(2¢ +1) = f(z) + 5a. When = = 0,(3f(1)- (0). We now form. the following table: 12 Problems and Solutions (A) f(e) = 2n+5 5 7 (B) f(@@)aa+l uf 2 (©) f@)=2-$ -¥) -3 (0) f@=2+} ; So, option (C) gives correct f. Ansier: (C) Is 3f(1) = f(0)? ‘No 19. A,B,C,M are four points on a circle such that: ABC forms an equilateral triangle and M lies on the arc BC as in the figure. Which c of the following holds good? (A) |MA| +|MB| + |MC| = 2|AB| f (B) |MA| = |MB| +c} (C) [MA[? = |MBI? + [Mo (D) [MA/?-+ |MB/? + |MC/? = Area of AABC Solution: Join B,M and C,M. Then being angles in the same segment, mZAMB: = mZACB = 60° and mZAMC = mZABO = 60°. Using cosine formula AB? = AM? + BM? —2AM - BM cos 60° and AC? = AM? + CM? —2AM «CM cos 60° Subtracting the second equation above from the first, since AB = AC and vos 60° = 4, we get BM*- CM? = AM .(BM — om) 1c (BM ~ OM)(BM + CM) = AM «(BM ~C™M). Either BM = CM ot BM+CM = AM. When BM =CM, M is the midpoint of the arc BMC and as A is the midpoint of the arc BAC, we have AM as the diameter of the circle. From Problems and Solutions 13 right angled AABM, BM = AMsin30° = 4¥, Similarly from right angled AAMC, CM = 4}¢, Hence AM = BM + OM. Thus in any case AM = BM +.0M. Answer: (B) 20. The positive riumbers « and y satisfy zy = 1. The mininum value of Jy + gir is (a4 8 (1 ©) Solution: sees} or 2 =}, we have a = =(?-4)t121 atqanwt gre ae) +121. Hence the inininium value required is 1 and is got when y? = gi or yt =}. Then 24 =2, Answer: (C) 21. A cubie polynomial P is such that P(1) = 1, P(2) = 2, P(8) = 3 and P(4) =5. The value of P(6) is (Az (B) 10 (©)13 (D) 16 For Solution , see page: 245 22. In AABC, the altitude from A to BC meets BC at D and the altitude from B to GA meets AD at H. If AD = 4,BD = 3,CD =2 then the length of HD is (a) 8 (8) § (©) v5 )§ Solution: Let BH produced meet AC in E From cycle quadrilateral DCEH, tan BHD = tanDGA ie, 22 4? 1,8. = 4 ‘1 ED DO’ “HD ~ 2 * HD=}. Answer: (B) 4 Problems and Solutions 23. A fair coin is tossed 10,000 times." The probability pot obtaining at least 3 heads in a row éatisfies’ (A\O.0. OSps Answer: (A) 24, The number of different Positive. integer triplets (a4) satisfying the equations 2? + y~ z= 100 and x+y —g-— 194 is (A) 0 (B)1 (C)2 (D)3 Solution: From the two given equations, on subtraction, we get y-stte-yam Problems and Solutions 15 or (y—2)(y+e—1) = 24. As ,y,z are positive integers, z+ y—1> 1 y-zc>Oory>ar..: & Since yt x and y— a are of the same.parity (i, either, both odd.or both.even) y+.2 —1 andy —« are of opposite parity. Also (y-+2—1)- (y—2) = 22-1 > 0. Hence y+ 2-1 > y—2 Considering the factors of 24 two cases are possible: @yts 8, y- r= Sor (i) yt o—-1= 4, yoo = Case (i) gives z = 3 and y = 6. These values of 2 and y substituted in the given equations give'two different negative values for z..Hence.case(#), is not admissible. €asex(ti). gives'e.= 1jg>=43- Then from both. the given equations we get 2'+ 57. Thus there is a unique solution. Answer: (B) Noteryta —1= 1g+e=2andy+o-1=6y-c=4 ate inadmissible cases as in these cases y +z —1 and y—z are both of the same-parity: i 25, If o(n) denotes the number of positive integers less than n and prime to n and"A'= {n|p(n) = 17} then the number of elements. in“A.isi (A) hifinit (B) 16 (C) 47 (D):zero Solution: For any integer NV, $(1V) is always even. Hence, the set A is empty. 5 Answer: (D) Note: If N =p? for some-prime p,.then the numbers which are nub.coprimme ton” are p,2p,3py: + :p"—!.p and, these account for .P°-1 pumbers. ‘Lhe remaining numbers in the set {1,2,-+- ,p"} will.be Jess than p” and, prime to, if. ‘These p" — p*-! numbers 16 Problems and Solutions must be therefore the number of riumbers in #(p"). e") =p" — pt =p D. As p— 1 is always even, ¢(p”) is even. If N has the prime factorization p{'p}?--“pf*, then $(N) = GOT" OPP) --- (Pf) and as each $(p*) is even, the product is also éven. Answer:(A) 2§. For what’ values off is an n-digit number uniquely determined from-the sim and product of its digits? (A) 1 (B) 2 @)3 (D) 4 Solution: For any n digited nuniber for all permutations of the digits the sum of the digits-and the product of the digits will remain the same. Hence the number cannot be uniquely determined unless n = 1. When n = 1, trivially, the number coincides with both product and sum. Answer: (A) 27. The number of polynomials P(x) satisfying the equation P(e?) + 2a? + 10a = 22P(z +1) +3 is (A)2 “(B)1 (C)3 {D) infinite Solution: Let P(x) be @ polynomial of degree n. Then the left hand side is of degree 2n in x while the right hand side is of degree (n+ 1). This means that n = 1, Let us take ‘P(a) = oz +6, Then ag? +b +207 + 1c = 2zla(x+1)+0)4+3 ie, (a+ 2)x? + 102 +0 = Qaz? +/2(a+ Be +3 This implies a = 2.and b = 3; Hence: P(z) is the polynomial 2x +3 and is of the firs degree. P(a).= 22 +3 is the unique polynomial satisfying the given identi Answer: (B) Problems and Solutions a7 28. Let 2 be. a positive integer. Then the number of common factors of n? + 3n+1 and n?+4n+3 is (A). (e+) (B) n-1 (C)2 (D).1 Solution: Tow positive integers p and q will have a common factor other'than one, only when, |p— 9| is 4 divisor of both p and g. Now, (ni? +4n+3) = (n?+3n+1) = n-+2 and itis not @ factor of # +4n $3 = (n+1)(nt 3) of n?+3n+1= (n+ 1P +n, Heneé the common factor of n? + 4n'4 3 and n? + 3n+1 is just 1, "Answer: (D) 20. If f(n +1) ="f(n) +n for all n> 0 or f(0) =1 then f(200) * equals . (A), 21100 (B) 21000 (C) 20900 (D)19901 Solution: As f(n+1) = f(n) +n, we get f(n+1)-f(n) =n. . Putting n = 0,1,2)-+- 199 in this equation successively And adding the two hundred resultant equations, we get after cancellations 7 (200) — f(0) =O+14+++.+199 = 199 x 2 $05 700 = 10900. “When f(0)= 1, f(200) = 19901. “30, The rays PX and PY cut off arc AB and CD of a circle with radius 4. If the length of arc CD is 2 times the length of the arc AB and length of CD is 4, then the angle APB is (A) 9° (B) 10° (c) 12° (D) 18° 18 Problems and Solutions Solution: Let O be the centre of the circle. Then OA = OB = OC = OD =4, . iy Length of the arc CD 4 implies that ZCOD = §. As arc AB = } arc. CD =, 2, LAOB = $. Let ZPAB =° 9: Then ZPDE = 8 (since ACDB is a. cyclic quadrilateral) From isosceles triangle OBD, ZOBD = ZODB. an Now Zope = 6-2 25 LODE = 3(n~ 4) and ZOBD = ZABD-i(n~*) i = 9+ ZAPB- 3. ; o~ = 0+ APB oe or LAPB = wag 1 ad RAMANUJAN CONTEST FINAL TEST - 2004 Find, all integers n > 1 such thet 2749 is en integer. Solution: We may write +3 __ In-3 mT tT So, Aad} is an integor if and only if Zee is an integer. Now Bet < G< 11 ifn> 7. Tabulating the values of 473 for ae, 2 ve <6 we get that for'n = 2and n= 5 only & met 3 is an integer being equal to 1, Thus 243 is the integer.2~ 1. =lor ~1=4 when n= 2 or n=5 respectively. A point is chosen on each side of a unit square. The four points form the sides of a quadrilateral with sides of lengths a,b, c,d. Show that 2s04 P44 Sd Wi. Assume that A,B and J" are fixed and C, D are variable points, 50 that the length of the segment C.D remains constant, Points X and ¥ are on the rays AC and BG respectively such that AX = AD end BY = BD. Prove that the distance between X and Y remains constant. Solution: We are given the quadrilateral ABCD is cyclic and therefore ZDAC = ZDBC = @, say. Then, ZDAX = ZDBY = 6. Comparing AADX and ABDY, we see-that the two triangles are similar since it is given that AD = AX, BD = BY. So, AD _ BD DX DY ZADB = ZADX — LBDX = ZBDY — ZBDX = ZXDY. Therefore, ADAB|||ADXY. This gives and ZADX = ZBDY DA _ AB Dx * X¥° : @ From the isosceles ADAX we get 10! DX =2ADsin 3 (2) 22 Problems and Solutions ‘As GD is of constant length, we see that 6 is a constant angle, From equations (1) and (2) we get XY = 2aBsin$ = constant . Y 4. In the adjoining figure, OB is the perpendicular bisector of DE. Ais a point on OB; AF is perpendicular to OB and EF intersects OB at C. Show that OC is the harmonic mean between OA and OB. ic, 2-0A-0B oc= OA+0B E 0. A B * Problems and Solutions 23 Solution: It is clear that AAFC is similar to ABEC. ‘Therefore : BE _ BC — =. 1 AF ~ AG a Again AAOF is similar to ABOD. So, ‘ DE _OB (2) FA OA From equations (1) and (2) we get BC _OB AG ~ OA OB-00 _OB oe OG- 0A ~ OA Therefore OA(OB- OC) = OB(OC - OA) or OC(OA+ OB) = 20A.0B 2.0A.0B Therefore oc= OAtOB’ . Find all integral values of 2, y, z,w given that cl + y] = 273” Solution: If z,y 2 5, then-5|z!+ yl. But § does not divide 27.3", Therefore x,y < 4. By. actually verifying for all the pairs.(x,y) € {1,2,3, 4} x {1,2,3, 4} we see that the solutions for the ordered quadruple (2, y, 2, 1) ate (0,0,1,0) (1,0,0,0) (0,1,0,0) (1,1,1,0) (1,2,0,1) (2,0,0,1) (@,2,0,1) (2,1,0,1) (2,2,2,0) (2,3,3,0) (8,0,1,2) (0,311) (8,2)8,0) (818,21) (4,4/4,1) . A convex polygon of nine vertices Fh, Pi, Pay+-> , Py is given along with six diagonals as shown in the diagram. We see that 7 triangles. PoP Ps, PoPsPs, PoPsPr, PoPrPa, PiPoPs, PyPaPs and P4PsPs are created. These triangles are to be numbered 24 Problems and Solutions A1,A9, As,+++ Ar 80 that P; is a vertex-of Aj. In how many ways can this be done? Justify your answer. Pi Py Po Pe Py Pp, Ps Solution: Ps Pa S.No | Triangle | PossibleA;s | Remark 1 | PPPs | Ar,Aa | from 2 Ag eliminated 2 | PoPPo| As,Ag _| from 3 Ag eliminated 3 | PoP6Pr | Ae,Ar _| from 4 Ay is eliminated 4 | AP Ar only possibility 5 | PAPPs | A1,A2,As | from 1 Ag is the only possibility 6 | PaPaPs | As,Aa,As | from 1,2,3,4 Ay is the only Possibility . T_| PePsPs | ards Ae | from 3,6 As is the only possibility Thus there is only one choice. Ai is APLPLPs, Ao is APLP:Ps, Ag is APLPsPs, Aq is AP: PaPs, As is AP4P5P5, Ag is AP PsPr, and Az is APyP;Ps- 7. Let f(e) be a linear function euch that f(0) = -5 and £(f(0)) = -15. Find all values of m for which the solutions of Problems and Solutions 25 the inequality f(a)f(m- 2) >0 from. an interval of length 2. Solution: Let f(z) = ax+b. Since it is given that f(0) = ~5, we see that b= —5. Also f(f(0)) = —15 gives f(—5) = —15 or ~5a+b=—15. So, ba = b+15 = 5 +15 = 10. ie, o = 2. Thus f(c) = 2-5. Let {| f(x) f(m — 2) > 0} = A. We want to find the values of m stich that A is an interval of length 2. We have i{2)f(m—2) > 0 <=> (22—5)(22 + 2m+5) <0 * 5 Im—-5 am-5 5 = ze (5, 2 ) ose (E85). "Therefore, length of the interval A = 2 if and only if 2 2 if i Solving the above equation we get m = 3 or m = 7. 8. In how many ways can you seléct two disjoint subsets from a set having n elements? Solution: Let X be a set having n elements, With every ordered pair of disjoint subsets (A, B) of X we denote a function fan from X to the three element set {0, 1, 2} as follows: 1, ifzeA fas(@@)=4 0, ifseB 2, if otherwise. 26 Problems and Solutions This association is in fact a one-one correspondence. So the number of ordered pair of disjoint subsets of X is equal to the number of functions from {0, 1,2} to X, which is equal to 3". One choice for (A, B) is (0,0). Therefore, the number of unordered pairs is 1 +1, On the other hand if selecting two disjoint subsets of X means selecting two disjoint non-empty subsets, then we have to remove the cases when A or B is empty. The unordered such pairs are 2" in number. Therefore, for disjoint non-empty subsets we have 3°=1419n — S19" 5, 2 as our required number of selections, RAMANUJAN CONTEST SCREENING TEST - 2005 1, The sum of digits of every possible 8 digit number is noted. Which sum occurs most: often? (A) Both 27,28 _(B) 41. (G) 32 ~— (D) Both 36,37 Solution: 1,00,00,000 and 9,99,99,999 are the first and Jast 8 digit number possible. Their sum of digits are 1 and 72 Tespectively. i.e. All other 8 digit numbers have the sum of their digits lying between 1 and. 72. We seek the most occurring number between 1 and 72. Since there are an even number.of (72) sums possible, its median aré the 2 middle most values which are 36 and 37. The distribution of the sums will.be symmetric and hence median and mode will coincide. The most occurring sum will be, therefore, both 36, 37. 1,2,-+» ,35,96,37,38,-+,72, Answer! (D) 35 Nos, 35 Noe, 2. 278-1 is exactly divisible by two numbers between 120 and 130: The sum of these two numbers is (A) 255 (B) 256 (©) 287 (D) 248 Solution: Factorising, 2-1 = (2-12 41) 4 (27 —1)(27 + 1)(2!4 4.2) (128 — 1)(128 + 1)(2'4 +1) = 127 x 129 x (2441) ke, (2% — 1) is divisible by both 127 and. 129 .which lie between 120 and 130. ‘Their sum is 256, Answer: (B) a 1 28 Problems and Solutions 3 3. & isa rational number satisfying (L—2)(L+e+2" +28 +24) = GY. Then l+zta?+oh+et+e% is (4) # ® # @ ©) % Solution: 3 (l-a)\Qlta+e2 423424) 33 2 orz= 1-2 _1-()% _ 6, _ Toe ep 1 2 gle "alr v lteter? +e3 pat p ed = Answer: (D). 4. Ifa tang +btany+ctanz = 1, then the minimum value of tan? x + tan? y+tan?z is (A) +042 (B) aokee (C) ab+be+ca (D) seitete: Solution: This is a problem of constrained minimisation using Lagrange’s multiplier method. Let f = (tan?2+tan?y+tan?z)+A(atenz-+btany-+etanz). For maximum (or) minimum $f =0=%o wt, os GL = 2tanesinat Aasin? © = 0or sin? 2(2tanz4-2a) = ie, A= MAE. Similarly \ = —248; and A= — Aung | Multiplying these values of 4 by a2, 62 we get A(a? + 0 + 6?) aoe mine _ cBtany — ~2tanz a+ a aeons and c? and adding, = —2(atanz + btany + ctanz) =—2, wk ¥ = a eo b “Mme = Serpe any = Se and Problems and Solutions 29 ¢ tone = Spey ae” Hence feteoh oe Pet an c= Tees rere) APO Answer: (B) 5. In the adjoining figure P,M,Q,R are collinear and PM = MQ = MS, SR? = PR-QR. Then (A) ZQSR= ZMSP {B) ZQSR= 2MSQ (C) 2QSM = 2PSM (D) ZSQR= ZSMP Solution: The given condition is SR? = PR-QR. M is the circumcentre of APQS Pp and ZPSQ = 90°. As SR? = PR.QR,RS is the tangent to the circle at S. + ZQSR = alternate AMPS = ZMSP (MS = MP). : : Answer: (A) 6. Two integers are declared equivalent if both are divisible by the same set of prime numbers, The number of non-equivalent positive integers less than 25 is (A) 9 (B) 10 (C) 16 (D) 17 Solution: 4,8,16 are equivalent (p = 2 is the common divisor) : 9,12,18,24 are equivalent (p =3 is the common divisor) 20,25 are equivalent (p= 5 is the common divisor). ., Number of non-equivalent positive integers less than 25 = 25— (8 +442) =16. Answer: (C) 30 Problems and Solutions If 2710 =0, then (r+ 1)(r +2)(r +4) is (A) integral (B) positive and irrational (C) negative and irrational (D) non real Solution: r?—r—10=0 implies r = 2¥2 is irrational. (r+ I(r +2)(r4+4) ‘This is irrational and positive for both signs. 1 a pT? + Mr +8 (7? =r —10)(r + 8) + 327 +88 0+88 +32r erate 8) (104 + 16/41) Answer: (B) 4h If 0° <2 < 90° and cosa = hy then logyg sinz+logyo cos z+ Jogi tanz is equal to Oe Bs Solution: (C)1 (D) -1 logio sine + logjg cos @ + logig tan = logio(sinzcosz- tan x) = logo sin? x = logya(1 — cos? 2) 9 = hogio(l ~ 5 = —logyg 10 = -1 1 = loga( 35) Answer: (D) Given that f is o function from R to R such that S(a +b) = f(ab) for all a,b and #(})=} find f (2005) (A) 2005 (B) Solution: f(z)= f(t +0) = f(@ x 0) = F(0). () 4 — (D) arabags = constant. But f(}) = }ygiven. Hence this constant = }. Problems and Solutions 31 Thus f(z) = 4 for all x ond iu particulur, f(2005) = }. Answer: (0). 10. A triangle has sides of lengths 6, 8 and 10. Find the distance “between its incentre and circumcentre . (A) vio (B) 25 () v2 (D) v5 Solution: Since 6? +8? = 102, © the given triangle ABC is a right . angled one as shown in the figure. 6 43) The circumcentre $' is the midpoint of the hypotenuse BC’ and réferred to AB, AC as axes of coordinates it a b is (4,3). 8 As in-radius r= 4 = } x6 x 8/}(6+8+10) =2, the in-centre I is the point (r,r) = (2,2). s1S = (E- IF OF = VB. Answer: (D) 11. The largest integer ‘n? which makes (“#4 also an integer is (A) greater than 470 (B) leas thai 460 (C) 21 . (D) greater than 460 but less than 470 Solution: Perform actual division to get (n+tP 484 nay 7 (21) + The greatest integer value of n, so that 484 is divisible: by (n +28) is 461. Then R.H.S will be an integer, ‘This the largest number 461 lies between 460 and 470. Answer: (D) Note: When n =.461, (n +.1)? = 462? and n— 21+ AS = 461 —21+1= 441, Qlearly. 441 x 484 = (462)?, 32 Problems and Solutions 12. If 2? +2+1= 0 , then 2 2 1\? LV? (4h) 44(e7 4b) i (=+2) +(#+3) +(2 +3 +2" + 7) is (A) 27 (B) 54 {C) 108 (D) 36 Solution: a Now = 14 yi=4_ -14 ivd : =m. cos( 2) 2 énin( a) = cos isin, say with 0 = z By De Moivre’s Theorem, ” = cosnd tisinnd a 1 Tet sq = wosnd Fisinnd. 8 a+ dD = 2cosné and (2” + a = 4.c0s7 nd. But cos § = 608 4 = co 70 = --- = cos 259 = cos 120° = —2. 0828 = cos 50 = cos8d = +++ = 008260 = cos 240° = —5 cos 38 = cos 68 = cos 99 = = cos 270 = cos 360° = .*. The given expression = 9(2.c088)? + 9(2.cos 20)? + 9(2 cos 30)? 1 1 = {-2 x 5)? +9(—2 x 5)? 49 x 2? = 54, Answer: (B) Problems and Solutions 33 Note: Even when 2?—2+1=0,since «= 144 we will have @ = , and we can argue as above and show that the given expression is 54. In this case 1 020 = 3, cos20 = -5, cos 38 = —1, cos 49 = — 1, co850 = 5 and cos 60 =1;--- Given expression 9(2.c0s 8) + 9(2.co8 20)? + 5(2.008 98)” + 4(2cos 69)? 0x (2% 298 + 9(8 x (—B))? + (2 > (1)? + 402 x? = 94945xK444x4=54 i] 13. If a, 30, 5a, 6, 6+3, b+5 are all roote of a fourth degree polynomial equation-where 0 < a < }, then the number of possible values of a is (a)o (B)A (©) 2 (D)3 Solution: 6,5+3 and b+5 are all distinct and positive. Therefore there can be only one more root ‘a’< 6. ‘Then Bo = b+3 and Sa =b+5 ora—-1=$ anda-1=$ which is not possible. So possible number of values of a is zer0. Answer: (A) 14. A triangle (non degenerate) bas integral sides and perimeter 8. Ifits area is A then A is (A) less than 2 (B) greater than 2 but less than 3 (C) greater than 3 but less than 4 (D) none of these Solution: The set of integer triples whose sum is 8 is {(1, 1,6), (12,8), (1,3,4), (2, 2, 4), (2,3,3)] . The set (2,3,3) alone gives a non-degenerate triangle. Then 2g = perimeter of the triangle = 2+3+3 = 8 and A=Arenof BA Problems and Solutions this triangle = /ale=ay@— 6) = VAA)G)A) = 2V2. Answer: (B) As 2<2V2<3,2 } implies n > 24% = 6684 ie, n> 669 aes < implies n < 295 = 1002} or n < 1002. Combining 669 6, the answer to our question is (-U-2) | We prove our claim by induction on n. We assume that the result is true for [X| 6, we have 14 3(n—3) < G@=N@-2) We have to check whether 2(3n —8) < (n~1)(n—2). Equivalently we have to check if n?-9n+18 = (n—3)(n—6) >0. This is clearly true for n > 6. So, in the case when each of the set in ¥ contains two elements of {1,2,3}, we are through. Let now assume that there is a set in § which contains exactly one element from 1,2,3. We may assume that this set is {1,4,5} . If all the sets in F contain 1, then |g] < =") and we are through again. Suppose now that there is one set in [| not containing 1. But this set has to intersect {1,2,3} and {1,4,5} and we may assume that it contains 2,4. We may take this set to be {2,4,6}. By induction hypothesis the number of sets in |$| not cqntaining 7 is at most =2K""8) since (rciyfaca) _ (n-ain-9) 2, If there are n—2 or less sets in |G| that do not contain 7, then we are through again!. So Problems and Solutions we assume that 7 belongs to at least n—1 sets in F. Bach of such sets must contain one of 1,2,3. If such a set contains 1, then it has to be {1,2,7} or {1,4;7} or {1,6,7}. If such a set contains 3, then it has to be {3,4,7}. This gives only six possible sets containing 7. So if n > 7 we cannot have n—1 sets in § containing 7 and we are through. It only remains to discuss the case when n = 7. In this case we compute the sets in $ that contain 6. We have now, the sets {1,2,3},{1,4,5}, {2,4,6},{1,2, 7}, (1,4,7}, {1,6,7},{2,4,7}, {2,5,7}, and {3,4,7} in §. Weare looking for sets in ¥ that contain 6 ad that intersect each of the above listed sets in $. They have to be {1,6,7} or {2,4,6}. Ifa set in ¥ contains 3 and 6 then it cannot overlap all the three sets {1,4,5}, {1,2,7},{2,5,7}. This means that the number of sets in F that contain 6 is less than n—2. Hence we are through by sets in $ that contain 6 is less than n—2. Hence we are through by induction. When n = 2005, the largest number of mutually disjoint 3-element subsets of X is 2004x2903 — 2007006 . . An infinite sheet.of paper is divided into unit squares by two families of parallel lines, which are mutually perpendicular. Some unit squares are coloured red so that every 2x 3 and 3x 2 rectangle contains exactly two red squares. How many red squares are there in a 9 x 11 rectangle? Solution: XX | XX XX | XX | XX RR [Xx ‘We claim that two red squares cannot share a common side. This is because, if they do s0, then look at the table above. Here row 2 and row 3 form a 2x 3 rectangle K and becawse of the given condition the squared marked XX in K are not Problems and Solutions 49 red. Again consider the 3.x 2 rectangle L formed by column 1 and column 2; the squares marked XX in L cannot be red. Now if we look at the 2x3 rectangle P formed by row 1 and row 2, we see that at best it can have one red square, violating the given condition. Thus no two red squares share & common side. R It is now clear that, for a red square, exactly one of the two diagonally adjacent squares marked XX must be a red square. It is also clear that all squares on that di: al are red and that tbe other red squares are those on every third parallel diagonal ine. Rm | xxpux] m | xx] xx] R [xx[ xx] Bl xx xx{ R | xx[ xx] R |[xx[xx|R | xx] xx[ R | xx | xx | R | xx | xx B {xx xx | R [xx | xx Rm | xx|xx[ Rr [xx| xx] pr [xx[xx| re [xx xx [xx [xx] pr | xx | xx xx [xx] Rk xx [xx [x [xx [xx] pf xx] xx] rR [xx [xx + R | xx | XX/ R |XX | XX| R | XX| XX] R | Xx — + xx} x | xx{xx| R | xx} xx] R | xx] xx] R xx | xx{ R [xx [xx] e [xx [xx] a [xx px We note that the first border column has exactly three red squares and the remaining 9x 10 rectangle can be partitioned into fifteen 3 x 2 rectangles each containing exactly two red squares. Thus we have 33 red squares in any 9 x 11 rectangle. RAMANUJAN CONTEST SCREENING TEST - 2006 1, The number of points (x,y) where x and y are real numbers such that, 2? + y? = 4, |z|-+|y|=1 is (A) 0 (B)1 (C)4 (D)8 Solution: If (x,y) is a solution then (¢,—y),(—z,y) and (—, —y) are also solutions. Let o20,y20. Then 2+y=1,and so 2?+(1-2)?=3, Le 22? —d24+4 = 0. This gives o = 4Y%,2-v3 ang =1-g= —YF 4 Thus we get 2 points (x.y) with 22> 0,y 20. The total number of points is 8. The answer is D. Note: The locus of the second equation is the square with vertices (1,0), (0, 1),(-1,0) and (0,—1). The circle intersects this in 8 points, 2 Let f,g : (0,00) — (0,00) be two fimctions where f is an increasing function and g is a decreasing function. Let h be a fimetion such that h(x) = f(g(z)). If h(0) = 0, then h(a) — h(1) is (A) always 0 (B) always negative (C) always positive (D) uone of these. Solution: Let 0 < & <-c0. Then g(:c) € (0,00) and F{g(2)) € (0.7). Further, 9(0) > g(x), as g is decreasing and hence f(9(0)) > f(9(a)), as f increases. Thus, F(o(z)) $ 4(9(0)) = (0) = 0 and f(9(2)) € (0,20). This implies that F(o(2)) = 0 for 0< e <0. Hence A(z) - h(1) =0—0, The answer is A. 3. Thice right triangles with the perpendicular sides (8em, 4em), so Problems and Solutions 51 (4om, Sem.) and (Sem, 3cm) are welded together so that the sides of equal length come together. How many cc of water will this hold? (A) 12c0 (B) 15ce (C) 10ce (D) 20ce Solution: The resulting A container is a tetrahedron whose base is a right triangle with perpendicular sides 3,4. The height of the tetrahedron is 5. So the volume is. 3(244) x § = 10. 4 ha The answer is C, 4, The equation z4—(r+1)x?+r = 0 has 4 distinct real solutions which form an Arithmetic Progression. The number of such 7 is (yO (B)1_— (C)2_— (D) infinitely many Solution: The given equation is (7? - F(z? — 1) = 0. The roots are real and distinct. So, r > Or # 1, and the roots are +/r,41. Herce, ~1,—yr, ¥7,1 are in AP; or —v7,-1,1, fF arein A.P. Thus, 2’= 3(2Yr) or 2V7 = 3(2); ie, -7 =} or 9, The answer is C. 5. The smallest: value that 2? — 7x +6 takes in the set {@ € Riz? — 2-30 2 0} is (A) 0 (8) = (C) 66 (D} none of these Solution: Required to find the minimum value taken by f(x) = 2?—7a-+6 as x varies in theset {x : (—-6)(+5) 2-0). ‘This is the union of the intervals (—oo,—5) and (6,00). Now, f(z) decreases for < J and increases for z > 1, So the minimum of f(z) in the first interval is f(~-5) = 66, and its minimum in the second interval is f(6) = 0. Hence the required minimum is 0. The answer is A. 52 Probleme and Solutions 6., Given that 2x? + 2y? = 16+ 12y +22, where x and y are integers, what is the smallest possible value that 4a —2y can take? (A) ~2 (B) -14 (©) 24 (D) 34 Solution: The given equation can be written as sigy?-8e-6y= jie. (x—4)?+(y—3)? = 36. Thus, (w — 4)? and (y — 3)? are squares of integers with sum 36. So they can be 0 and 36 only. The solutions are given by z—-4=0,y-3 = +46, or r-4 = +6,y—-3 =0. Hence, (x.y) = (4,9), (4,-3), (10,3) or (~2,3). The corresponding values of 42 —2y are —2,22,34,—-14. The minimum is —14. The answer is B, ay . If x,y,z are nonnegative integers so that s+y+z = 12, then the maximum value of zyz+ zy +yz+22 is (A)115 (BY) 112 (C) Ht (D) none of these Solution: We have (z+ 1)(y+1)(2+ 1) = ayz+sy+ peteototyt+ e+] = ryz+oyt ye +20 +13; and (e+1)+(y+1)+(z+1) = 15. Using AM.GM inequality we get ayztaytyztsn+13 < (2)> = 125, syz-+aytyz+er < 112. Equality holds if 2+1=y+l=z+ljieifs=y=2=4. So the desired maximum is 112. The answer is B. 8. Three non-collinear points A,B and C are given in space. The number of lines I in space such that A,B and C are equidistant from J is (ayo (B) 3 (o)6 (D) infinitely many Solution: The points A,B,C determine a plane, Let A’,BC’ be the midpoints of BC,CA,AB respectively. Consider the plane II perpendicular to the plane ABC and containing the line B’C’. Let ! be any line in If and parallel to BIC’. The points A,B and C’ are equidistant from the line B/C’. Problems and Solutions 53 Using this we can see that they are equidistant from J also. ‘There are infinitely many such lines in II. So the answer is D. 9. Ina AABC, ZA = 30°, AC = 10 units and BC = 4 units. Which of the following statements is correct? (A) 2B is acute (B) 2B is obtuse (C) 2B is 90° (D) no such triangle exists For Solution , see page: 246 10. The number of solutions (,y) where a and y are integers, satisiying 22? + 3y? + 2c +3y = 10 is (A) 0 (B) 2 (c)4 (D) none of these For Solution , see page: 246 11. The range of the function f(r) = (7—r)P,-3, where 7 is a non-riegative integer is (A) (,2,3,4,5} (B) (1,2,3,4} (©) {1,2,3} (D) {4,2} Solution: The domain of f is given by 7—r >r—3>0, ie, 3<¢ 1 <5. So the domain is the set {3,4,5} and hence Range= {f(3), f(4), f(5)} = {4P0,3Pi,2P2} = {1,3,2}. The answer is C. 12, If the graph of the function y = f(x) is symmetrical about the line « = 2, then (A) f(z) = ~-f(-2) (B) fQ+2) = f(2-2) (C) f(z) = F(-2) (D) f(z +2) = f(x ~2) Solution: The condition of symmetry about the line 2 =2 is hoe F (ar) = f(z2) whenever x; —- 2 = 2~22. The answer 54 Problems and Solutions 13, The position number of the permutation 443321 when the numbers formed by the digitsl,2,3,3,44 are written in increasing order is (A) 720 (B) 180 (C). 120 (D) none of these Solution: The total number of permutations of 1,2,3,3,4,4 ie gr = 180. The greatest of the number formed is 443321; its position is the lest,i.e; the 180th. So answer is B. 14. We consider three edges of a cube, all’pointing in different direction, no two of them having common vertices. The plane that contains the three mid points of those ciges intersects the cube in: (A) An cquilateral triangle (B) A square (C) Two different polygons are possible (D) Regular Hexagon Solution: Each edge of the cube is the intersection of two faces. So the three chosen edges determine 6 faces. If w face meets two edges, then these two edges are parallel or intersecting in a vertex. It follows that no two of the three chosen edges can have an common face. In other words, tL the 6 faces determined by the three edges are different. ‘The plane containing the midpoints intersects each of the 6 faces of Problems and Solutions 55 the cube in a line segment. Hence we-get o hexagon in that plane, The answer is D. 15. The function f ;N— N defined by f(n) = [¥7] where N is the set of natural numbers and [z] is the greatest integer Sa. Then f(1) + f(2)+--- + f(200) equals (A) 625 (B) 615 (C) 525 (D) none of these Solution: Clearly, f(x) = [VZ] =k for z then Z=[2,2+1) and X=(z+1,2+2] ‘Thus in either case, X and Z do not overlap, So k= 0. The answer is C. 56 18., 28 ~1 is exactly divisible by some numbers between 120 and 130, The sum of such divisors is: (A) 255 (B) 256 (C) 257 (D) 258 Solution: We have (28 ~1) =, (2! — 1)(2"4 4 1) = (27 ~ 1)(2” + 1) - 16385 = 127 x 1295 x 29x 113. This is not divisible by an even number. Further, this is not divisible by 11, 41 or 25; hence, it cannot be divisible by 121,123 or 125. So the only divisors of 278 — 1 lying between 120 and 139 are 127 and 129 whose sum is 256. The answer is B. 19. A hoop is resting vertically at stair case as shown in the diegrem. AB = 12cm and BC = 8em. The radius of the i hoop is (A) 13 (B) 12v2 es (C) 14 (D) 13V2 Solution: Let ZCAB = 6 and P be any point on the major arc AC. Then ZAPC = 6, and hence AC =2R:sin@ where R is the radius of the circle. Thus, 20) | Re AG? _ 148? 2BC” 16 As: ‘The answer is A. For Aliter Solution: Let BC meet the circle again at D, and let M be the mid point of CD. Then the radius = BM and BC.BD = BA. Hence BD = 127/8 = 18; radius = 2418 = 13, The answer is A, = Problems and Solutions 57 20. - Two circles’ intersect at P,Q. ABCD is a line drawn meeting the circles as in the diagram, Then oor (A) ZAPB + ZCQD = 90° ) (B) ZAPB+.20QD = 180° (C) ZAPB = ZCQD eS (D) ZAPB~- ZCQD = 90° Solution: Since the quadrilateral PQCB and PQDA are cyclic we get ZAPB = ZAPQ - ZBPQ = ZAPQ- ZQCD = 180- CDQ - ZQCD= ZCQD. So (C) is true, answer is C. Note: Since (C) is true, (D) is false. Keep the smaller circle and P,Q,B,C fixed.Replace the other circle by a bigger one. We can see that ZAPB+¢GQD cannot remain constant. So (A) and (B) are false. 21. In the adjoining figure AC = 2006, ZACB = @,BDLAC, DELBC,EPLAC. ‘Then A BF = >, (A) 2006 sin*@ (B) 2006 sin? @cos@ (C) 2006 sin? @ cos? ele ‘ (D) 2006 sin cos? @ Solution: BC = ACcos@, CD = BCcos@ = AC cos?0, CE = CDcw@ = ACco?@, EF = CEsin@ = AC sin 8 cos*@. The answer is D. 22. A point O moves inside.2 square ABCD so that ZAOB + ZCOD = 180". As O moves, it traces out (A) The perimeter of the square (B) the whole of the interior (C) only the centre of the square (D) the diagonals \ 58 Problems and Solutions Solution: Let O be any point on the diagonal BD. In triangles AOD and COD we have ZADO = 45° = ZODO, AD = CD,OD is common. Hence they are congruent. 4 7 ea ) B c Hence, ZAOD = ZCOD, £AOB+ZCOD = ZAOB+ ZAOD = 180°. Similarly, if-O is any point on the other diagonal AC then ZAOB + ZCOD = 180°. We shall prove thet if O is any point inside ABCD which does not lie on the diagonals than ZAOB + ZCOD # 180°. Suppose as in the figure, that O lies in the quadrant containing AD. Then ZAOB = 180° ~ (45 +0 +45—p) = 0+p-z, 2COD = 180°—(45+4+45—y) = 90+-y-q, ZAOB+ZCOD = 180° + (p+y—z—q). If E is the foot of perpendicular from O to AG, then OC? — O# = CE?— EA? > 0. So OG > OA. Similarly, OB > OD. If follows that z > y and q>p. Hence, r+qg>pt+y, and ZAOB + ZCOD < 180°, We can deal with the remaining cases similarly, So the locus of O 8 is ACUBD. The answer is D. 23, A box contains 900 cards, each having different integer between 100 and 999 written on it, Mary draws randomly & Problems and Solutions 59 number of cards from the box and computes for each card, the gum of its digits. How many cards will Mary have to draw, to be sure that at least 3 cards give the same sum of digits? (A) 53 (B) 54 (C) 55 (D) 56 Solution: Let s(n) = sum of digits of n. Then 1 < s(n) <27 for 100 n= 20. This implies that 21 22 100 [rosie] + [r+] +o p+] -o But the terms in LHS form an increasing (nondecreasing) sequence of '0's and ‘l's, (since f+ 24... are all <1) Hence 64 Problems and Solutions 65 i= [1 iol [r+ al- [r+ #\- [s+ a - [+3] - [r+ and bP =0. This means that [f + $4] <1= [f+] 94 f+ig< iste or 6 0 ms! 2005 + 5 < 100r < 2006 + Fir => [100r] = 2005 or 2006. The interval for 'r" is iia by oe 2557 <7 <2 Ti 2. ABC is an equilateral triangle of side 1 unit. A line I is drawn through A out side the triangle 7 ABG, Circle C,(O1,71) \Y touches the _ lines Ny 7% 1,BG and AB and circle 2d C2(Oa,r2) touches the linea 1, BC and AC. For different positions of the line 2, prove that (r1-++ra) is a constant. 66 Problems and Solutions Solution: Suppose Z0,AM = ZOyAL = 6 Then Z02AL, = ZO2AM; = 60% ~ 0 (00 the figure). x B 7 Cc Ki Also 20, BL = £0,BK = 60° = £0,CL, = Z0.CKi BL = 1, cot60%, AL = ry cot 8 Qa) 1 = AB = AL+ LB = ri(cot0 +-cot 60°) (2) Similexly, 1= AC = Aly + L1C = ri(cot(60” — 8) + cot 0°) (3) From (2) and (3) 1 1 Ti +T2 = SotG0" + cotd + cotGO + eotlGNr a 8) _ sinG0°sing _ sin 60° sin(60° - 4) ~ sin(60° +6) "~~ sin(i20° — 6) _ sin 60°{sin@ + sin(60° ~ 9)} ‘sin(60° + 6) _ ¥3 {sind + sin(120° + 6)} 2 sin(60° + 6) V3 2sin(60° +4) cos60° V5 =F ia(ee a) — pe constant. Problems and Solutions 67 Aliter: Suppose that ! meets BC at D, Suppose , a8 suggested by the figure, that D is on the right of C. Then Ci, ig an ex circle of AABD, and Cz is the in circle of AACD. Let A, 41,42 be the areas of triangles ABC, ABD,ACD respectively. Let CD =z and AD = y. Then 52 = =. 1 Hence Ay = 2A,A1=A+A2=(e+1)A, mee PAL g@+DA Ay 2a \"Teaty-l- aty 2 Ite+y otytl Thus mtn _ z+l z 24 ogty zt+ytl a4eytotatytl+c2+zy (e+yP+ot+y By cosine formuls, y? =a? -+1+2. So the numerator of the above expression is-2? + 2zy + y? + 2+y =the denominator. Hence ry +1 = 2A = 8, If D is on the other side, we can simply interchange B and C. If 1 is parallel to BC, then 7) = 12 and mr, +79 = the altitude of ABC = 2. 3. Let AyAg'+ Ar, Bi Ba-++Bz,CCa++-Cr be regular heptagons with areas S4,5g,Sc respectively. Let AjA, = 8, B3 = . C1C4. Prove that 5 < Peake «2-2. Solution: Let A)4g = a; Aids = 6; AiAg = c. Now, AjA3AqAs is a cyclic quadrilateral. Hence by Ptolemy’s theorem, Ai Ag: Ags + Ais + AsAg = Ar Ag: A3A5. bot cam ber +S =1 (1) 66 Problems and Solutions Now, 44,4243 and 4B; BBs are similar. B,Bz a aB, Bs —Ss=— BB, = BiB, bt (By Bs = AiA2) () Also, 4A,A3Aq and ACiCsC; are similar. CaO, _ Asay _a@ CiCy AlAg ec a? or C3Cy = = (3) From (1), (2) and (3), we get Sp+Sc _ Sa, Sc _ (By (AG)? fs 2 a SA Sa Sa (ArAg)** (Aya)? > a? ta? > BS ao ag la ay Now Ft S> apts =5- wuality not possible as 2 2}. Thi 1 Sp+So_ FEquality not p 72 This proves 5 <—"G— Problems and Solutions Now, SatSo _ ,_ 20? a? __ sin? So be? bc sin sin F 7 sin? § = Fun™¥ cost Foes 8sin? § cos? F cos F 1 6 since coe 2% < ops =3= 40 v2 7 S, 69 1 ~ Hitec Feo ® ” B04 8) v2-1 =: a Aco, $8450 3-28 <1 -(/8-1) =2- v3. fa Thus 2 < 884 5¢ go Va, 2< Sa 4, Prove that there are infinitely many ordered triples of positive integers (a,b,c) such that ged (a,b,c) =1 and a?b?+b3c? + ca? is a perfect square. Solution: Choose two positive integers z,y that are relatively prime and z being an odd integer. Let a = x9,b = 2y?, and e=ay. Then ged (a,b,c) =1 Now 7b? +c? +c4a? = at dy’ + dy’ daty$ + dx2y® + 26y3 (2ey8 +25y)? perfect square i a a A%y? 4 yet Thus, there are infinitely many such triples (a,b,c). 70 Problems and Solutions Aliter: If c= a+. then, ab 4 ct +a? = a2? +P(at+b)? + (a +b)a? = at +205 + 30%? + 2ab8 + of = (+ab4+b)? = perfect square We just have to choose a,b such that (a,b) = 1. Then ged (a,b, a +b) = ged(a, b,c) = 1. 5. Let A bea k-element subset of {1,2,3,++-,16}. It is known that every two subsets of A have distinct sum of their elements and for any (k +1) element subset B of {1,2,3,--- ,16}. containing A, there exist subsets of B with equal sums. a) Prove that k <5 b) For different subsets A with the given property find the maximum and minimum possible values of the sum of the elements of A. Solution: There are 2" subsets of A where A isa k-element subset of {1,2.--+ ,16}. If any two subsets of A have different sums of their elements, then the sum of the elements of the set A must be at least 2*—1. : [Note that A has 2*--1 nonempty subsets, each having 2 different sun]. Case (i) k> 7 Then 2—1 = 2424-41 = 16x 2'-4—1 > 16k The maximum sum of a k-clement subset is 16-4 154-416 — (k~ 1) = isa — 4] < MOBO ase So the sum of the elements of A satisfies 16k < Si, < 13k for k'27. This is impossible. So k <6. Case (ii) k = 6 Consider the 1 clement, 2 element 3 element and 4 clement subsets of A, These account for (6) + (@) + Probleme and Solutions 71 (8) + (8) 9 6415 420+ U5 = 86 differerit subsets of A. ‘Tho minimal sum of gueh a subsot is Land tho maximal sum is 1G -+ 16 4 1d 4 12 = 67 (not it cannot have 13, as this would give 16419 « 18-414), Supposo 1 € A then the maximal sum in 16-4 14-+12++ 10 = 62 (note: 1 € 9, 16 € S: will moan 18 ¢ S ate), So fur the 56 different subsets of A listed above, there aro only 52 different possible sums. So, two subsets will have tho same sums, Hence 1¢ A. Suppose 2 € A, then the maximal sum is 16-+15-+124-11 = 54, This means that there can be only 53 distinct sums, So 2¢ A, If 1 ¢ A, 2¢ A, then the possible sums are in the interval [9,57] and this intérval contains only 5 integers; and we have 56 distinct subsets. So k £6 or k <5. This answers the first part of the question. Claim 4: For any such set A= {a1,02)°++ ,a¢} a1 -Fag-beo + ay > 16. Otherwise B= {a1,02)+++ ) dp; 16) will have all subsets having ifforent sums. ‘This contradicts our assumption on A. 6) bag te tag 2 16. Claim 2: If A= {1,2,4,9} then A satisfies our requirements and its sum of the ¢lements= 1+2+4+4+9 = 16. So, the minimum sum of an allowable A is 16. Glaim 3: A = {9, 12, 14,15, 16} satisfies our requirements and’ its sum =9-+12-+14+15 +16 +66. Suppose A is an allowable subset with its sum > 66. If 16 ¢ A, then the maximum sum that A can have is less than 15+ 14413412411 =65 < 66. Therefore, 16 € A. If 15 ¢ A, then the maximel sum is less than 16+ 14+13+12+11 = 66. So 15€ A. If 14 ¢ A, then its maximal sum is less than 16-+ 15 +13 + 72 Problems and Solutions 11410 = 65 < 66. So 14€ A, Thus {14,15,16} cA. Gleatly 13 ¢ A, since 16413 = 15+14. If 12 ¢ A, then the maximal sum is less than 16+ 15-+14+ 11+ 10 = 66. ‘Therefore, 12€ A. So {12, 14,15, 16} CA. It is checked that the fifth element has to be 9, Thus A = {9,12,14,15,16} with the maximal sum being 66. 6. Let m be areal number, such that the roots x, and z3.of the equation f(z) = z?+(m—4)z+m?—3m+3 =0 are real numbers. a) Find all values of m for which 2? + 23 b) Prove that 1< P24 Misc mM. Solution: We have a+m, = —(m-4) a) weg = m'—3m+3 As x1,%2 are real, (m— 4)? — 4(m? —3m+3) 20; ie. 3m? ~4m—4<0 m ie, (3m +2)(m—2) <0 @) ie, emer (3) atta} = 6 => (m—4)?—2(m?-3m+3) =6 or m242m—4=0 @ Therefore, =-14 V5 (8) (8) and (6) give m= -14 vB. (b) may, mB maf(1 — a9) + ma}(1 — 21) Imm" 1~m (= 21) = 22) ae , = m—6m+1_ using (1) ma}, mag i—m “is +8 = m?-6m+9=(m—-3)’, Problems and Solutions 73 In #2 < m< 2, (m—3)? is monotonically decreasing and (mm —3)*hmm-a/s = 32 and (m= 3)"|mar = 1. Hence 1<¢ Mt 4 8 4g ¢ 2, 1l-m 1l-m 9 The middle member of the inequalities in (b) is not defined when #1 =1 or 2 =1. This will be the case if m=0 or 2. So we need consider the values of (m—3)? for -3 1 for such values of m. 7. Find all functions f ;R + R with the property Fafa) + Fy) = (F(@)? + Solution: Let z= 0. Then we get SUF) = (FO +4, Vy ER (1) Let y= —(f(0))?. Then f(f(—(7(0))?)) = 0 , There exists 6 €R such that f(2) =0 (2) Let z=, Then we get Kfw)=¥ WER (3) Replace by f(z) in the given functional equation to get SF(=)« FF (@)) + FO) = FE@)? +9 or f(xf(a) + f(y) =2? +y for all z,y in R 4) So we get (f(z))?+y=27+y forall z,yeER => (fe) =2?, WeeR 5) (FQ)? =1 or f(1) = 41. Case 1; f(1) =1. Then f+ f(y))=1+y WER. 6) 74 Problems and Solutions From (5). (1+ f(y)? =G+y), WER. 7) (7) with (5) gives f(y) =m WER. Case 2: f(1)=—-1. Then f(-14 f))=1+y WER. (8) From (5), (f(y) - 1)? =(1+y)?, WER. (9) (9) with (5) gives f(y)=—y, Wye R. So there are only two solutions f(a) =2, VeeR, f@)=-2, VWweR. 8. For the subsets A1,42,-++, Azoos of a set M, the condition [Ai] > ${M| for i = 1,2,-+-,2006 is satisfied, where |X| means the number of elements in X. Prove that there exists an a € M which belongs to atleast 1505 of the subsets Ayi= + ,2006. Solution: Assume that the result is wrong. Then each « € M belongs to at most 1504 sets Aj. = 1,2,--- ,2006. Then H(z) }le € Ai,t = 1,2,--- 2006} < 1804,M] (1) From the hypothesis Geshe € Aiyé = 1,2,---2008}| > $006,104] = S28 a. (2) From (1) and (2) 2992|M} < 1504)M or 3009 < 3008 which is absurd, Hence Je ¢ M such that belongs to at least 1505 of the subsets Ai,é = 1,2,--+ ,2006. RAMANUJAN CONTEST SCREENING TEST - 2007 1, With how many zeroes does the product of the first 2007 consecutive prime numbers end? (A) None 1 (2 @)9 Solution: The first prime number is 2. The third prime number is 5, And no prime number divides the other prime exactly. Hence, the product of first 2007 prime numbers ends with exactly one zero. Answer: (B) 2. The perimeter of the shaded figure made out of 9 unit squares is equal to 20. What is the greatest number —| of squates we can connect to the shaded area so that the perimeter of the new figure would still be 20? (A)16 (B)7 (C)18 (D)W Solution: Minimum area of square blocks for perimeter 20 is 5? = 25. The shaded region can be embedded in a square of side 5, .", 16 squares can be added. Answer: (A) . There are five cards on a table as shown in the picture. Every card has e letter on one side and a number on the other side. s 75 6 Problems and Solutions Jacob said, Each card on the table that has a vowel on one side has an even number on the other side. Alicia wants to find out if Jacob is saying the truth. What is the smallest number of cards that she must turn over to find out? (At B)2— (C3 DY A Solution: It is enough to turn 2 cards A and 7. The given condition is one way only, not if and only if. Answer: (B) 4, The ratio of the sector’s radius to the inscribed circle’s radius is 3:1, What is the ratio of their areas? (A) 3:2 (B) 4:3 (C)V3:1 (D).2:1 Solution: We note first that the angle of the sector is 60° when the radii are in the ratio 3:1 SESE cal Ss 60° Ratio of Areas = % 7 * (Br)? axe 1 3 Tg tee Answer: (A) 5. The picture illustrates a rectangular prism. The lengths of the prism’s RS / face diagonals are |XZ| = 58, A |XY| =8 and: |¥Z| = 9. What is the length of the prism’s diagonal AX? (A) v90 (B) 10 (C) Vi20 (D) 11 Problems and Solutions 77 Solution: Let the length, breadth and height of the cuboid be 16,4 units respectively. Given: 0? +A? = 55; [7+ 0? = 64; 4 Pal PLPGP LAER EP = 64455481 > 22 4R+h3) = 200 AX=JP+P +R = 10 Answer: (B) 2 “The picture illustrates a church stain glass. Letters R,G and B represent glass of red, green and blue color, respectively, The total area of green glass is 400cm?. How -many cm? -does the area of the blue glass have? (A) 120 7 (B) 90. V2 (©) 396 (D) 400 Solution: Let the radii of the bigger circle and the smaller circle be r, § (cm) respectively. 4R+4G+4B = i? (Q) r AR+8G = 4-4(5)? (2) 4B-4G=0 But 4G = 400cm? = 4B=400cm?, Answer: (D) 78 Problems and Solutions 7. Segments starting with points M and N and ending with vertices of the rectangle ABCD divide the figure into eight parts (see the figure). ‘The areas of three parts of the rectangle are indicated in the picture, Whet is the area of the shaded region? (A).20 (B).21 (CG) 25 (D) 26 o c Solution: Let the required pI area be given by z as in the figure. Consider the indicated areas 2,y in the figure. aN i“ 1 Areaof AAMD = $x AMxAD=3+2 4) Area of AMBC = 5X MBXBC=+y42 2) (1)+(2)= 4x ABX AD =25+2+y as AM+MB = AB and AD =BC. But area of AADN = } x AD x AB, as AB is the height of the triangle. wGtety=B+rt+y 22525 Answer: (C) 8. For what value k, the number V24V3- 24 Vi+v3- Y2+ 2+ V24V3 - 2-24 V2+ Vi is the solution Problems and Solutions 79 for the equation 32‘ + 5x + 2k = 07 (A) -5 (B) -4 (c)9 (D)2 Solution: (2+ 2+ V2+ V3): (2— 2+ V2+ V3) =H ~ (24.724 v3) =2-2+V5. Now, (2+ V2+ v3) :(2- ¥2+ V3) =P-2-V3 =2-¥3. Also, (2+ V3) -(2- V3) =4-3=1 . The given arithmetic expression reduces to VI = 1. For 1 to be a root of 324+ 52+ 2k = 0, we have (8 +501) +2k=0 3 ka—4, Answer: (B) 9. Manas removed one number from the sequence of ten consecutive naturel numbers. The sum of the remaining nine numbers is 2007. Which of the following numbers did Manas possibly remove? (A) 218 (B) 219 (C) 220 (D) 221 Solution: Let the 10 consecutive natural numbers be art let 2 e+3,...,2+8 249, Sum of the 10 consecutive natural numbers = 102 + 45, a multiple of 5, ending with digit “5”, 80 Problems and Solutions If sum of 9 remaining natural numbers =2007, the left out number should end with digit ‘8’. Answer: (A) 10. What is the least possible value of 21z + 14y, if zy = 6 and y>0? {A) 48 (B) 83.5 (C) 84 (B) 96 Solution: By A.M—G.M inequality, we get Bet > Sta iy = 7, /ozy = 42. Note that zy=6 and y>O>2>0. *, 21a + l4y > 84. ie., the least value of 21x + 14y is 84. Answer: (C) 11. The picture illustrates a 4 5 regular hexagon with the side length equal to V3. Quadrilaterals XABC and QPXR are squares. What is the area of the shaded triangle a CPSs? (a) =) Ge Of M4 Solution: In a t g— regular hexagon, each interior fa \ VA Rm angle = 120° j ALD « ZARX = ZX AR = 30° a => LAXR = 120° => ZPXC = 60°. Problems and Solutions 61 Also RX=AX > RX=RQ=QP=PX=X0=X A=AB=BC. ZPXC = 60° = APXC is an equilateral triangle. By cosine formula, we get AR? = AX? 4 XR? —2AX « XR-cos 120° 3 3= 242 — 2(-}) = 31? =3 (or) t= AX =XR=1 unit, 2. PCG = 1 unit Altitude on AR in AARX = } units. Altitude on PC in APXC = % units. % Oo oy “oy Figs ZKAS = 30° > SAL AR. SA? = 3.43—2V3V5- cos 120" = 9 SA=3 units. <. Altitude on PC in APCS = (6 -}- ¥) units, Area of APCS = Product of 5x -}-4) sq, units, Answer: (A) 82 Problems and Solutions 12, A pentagon with area 40 has equal sides but not necessarily equal angles. The sum of the five distances from a point inside the pentagon to the sides of the pentagon is 16. The side length of the pentagon is 5 5 A) 5 Bq (G) 12 @)5 Solution: Area of pentagon = RX 2(atb+ctd+e) 1 = 4= 3 xzx 16 sab. Answer: (D) 13. If [a] stands for the largest integer less than or equal to a, (For example. [2.7] = 2, [-2.7] = -3), then the set of solutions of the equation [2 - }] = [2x+ 4] in the interval (0, 1] is 2 4&3 G2] © we Solution: 0< <1 .5'<2-}<} Also 4 < 2n+}<23 1 3 [--3]-0 or ~1 and [22+] =0, 10r2 Hence [=-4] 7 [ze 3] > both equal 0 ie, OS2—}<1 and O< 24) <1 he, $a <§} and Bser<} (or) }Sa2<1 and 01>0. As 6b-6<6b~3 <6b+2< 6643, the greatest fraction is Answer: (A). Problems and Solutions 85 18, The number of pairs (x,y) where both x and y are.real satisfying 2? + y7+2=(1+2)(1+y) is (A)o (B)1 (C)3 (D)4 Solution: Given 2?7+y?+2=1+2+y+zy. Multiplying both sides by 2 and rewriting, we get (2—y)?-+(z—1)?-+(y-1)" = 0. As x,y are real, this implies s—y = 0, e-1=0 and y~1=0 ie,, @=y= 1 is the unique solution. Answer: (B)’ 19. The number of integers x for which «, 10 ond 24.are the sides of an acute angled triangle is (A)2 (B) 3 (c)4 (D)9 Solution: For an acute angled triangle, P 24>10 P 24 a>14 ‘2 can take integer values from 15 to 25. Only 25 > 24. Hence, there is only 1 value of = in this case, 86 Problems and Solutions . 2 >a > 10 one ante 24? 2>22 But 2< 24 Hence < can take velues 22, 23. ‘There are two possibilities in this case. Answer: (B) Note: Also when 2 = 24, we can have an isosceles acute angled triangle, 20. The number of two digit numbery'n such that the difference of n and the number formed by reversing the digits of n, is prime, is (A) 10 (B)7 (©) 4 (D) 0 Solution: Let the 2-digit number be 10+ ywhere x,y are the digits in the order. Now (102+ y)—(10y +2) = 9(e—y), is always divisible by 9 or otherwise zero. Hence it cannot be a prime. Answer: (D) 21. In triangle ABC, A is (0,0), B is (18, 21) and C has integer co-ordinates. The minimum area of the triangle ABC is (A) 3 (B) § (C) 3 (D) none of these Solution: Let the integer co-ordinates of C be (x,y). Area of AABC = -&3| 5l0(21 ~ 9) + 18(y ~ - 0) + 2(0 — 21)] =p 5li8y — Qa] = +5 3 6y- 72) Problems and Solutions 87 ‘The smallest difference of (6 —7z) for integers z,y is 1. Answer: (A) Note: When (z,y) is (6,7), we have 6y — 7x is 0 but,, then; A, B, C will be collinear. 22. For n € N, where N is the set of natural numbers, if a sequence dp is defined as a; =2,dn41 = an + 2n,(n > 1) then the number of digits of azyy7 is (A) 2007 (B) 2006 (8 @)7 Solution: 2007 = 2+ 2[1+2+3+... +2006] = 2+ 2006 x 2007 = 2+ 4026042 = 4026044, a 7-digit number. ‘Answer: (D) 23. The number of ordered triples (a,b,c) of positive integers which satisfy the simultaneous equetions ab-+ lc = 44 and ac+ be= 23 is (A) 3 (B)2 (c)1 (@)o Solution: Given b-(a +c) = 44; c-(a+b) = 23 where a,b,c are natural numbers. c must be 1 as a+ >2 and 23 being a prime number. “*, The given equations reduces to b-(a+1)~ 44; o+b=23. ‘The solutions are: b= 2, a= 21 and 6=22, e=1. Answer: (B) 24. x,y are real numbers satisfying the condition (x — 2)?+ (y—2)? = 1. The largest value of (2) is (ay 2£YE wy Sto) $4 non of thre 88 Problems and Solutions Solution: Let s=2+sin@; y=2+cos0. => cos = 2k -2+ksind Squaring and simplifying, we get (#? +1) sin @ + 4k(k — 1) sind + 4(k - 1)?-1=0 Converting into inequality, we finally attain 3k? — 8k +3<0 _84VG=—4xSxS 4tv7 44v7 Ph ee) Answer: (B) 25, A right angled triangle has to be constructed with the legs parallel to axes of co-ordinates. If the equations of the medians to the two legs are 3r—y+1=0 and mc—y+2=0, the number of values of m for which such a triangle exists is (A) 5 (B)3 (C)2 (D) none Solution: Let the three vertices be A(a,), B(a,b+¢),C(a+d,d). Let y = 32+ 1 pass through B and the mid Point of AC(a+d/2,b). ‘Then we get b+e=3a41 a) b=3(a+d/2)41 (2) Froin these wo get c= —3d/2 (3) Let the other median y = mz-+2 pass through C and mid pint of AB(a,b + c/2). Problems and Solutions 89 ‘Then we get b= m(a+d)+2 @) b+c/2=mat2° (5) From these we get o/2=—md (6) From equations (3) and (6) we get 34/4 = md m= 3/4. If we take the other way that y = 3x +1 passes through C and the midpoint of AB and y.= ma+2 passes through B and the midpoint of AC we get m=12 ‘Thus there are just two values for m. Answer: (C) 26. Which of the following is correct? (A) The ratio of the three altitudes of a triangle are in the ratio 1:2:3 (B) The circum-centre of a triangle always lies inside the triangle (C) The angles of quadrilateral are in the ratio 3:3:2:4. Then two interior angles of the quadrilateral are 90° (D) ‘The in-centre of a triangle need not be within the triangle. Bolution: Answer: (C) 27. If the product of the digita of a four digit number is 72, then the maximum possible sum of the digits of the number is (A) 12 (B) 25 (CG) 22 (D) 19 90 Problems and Solutions Solution: The 4-digit number can be constructed from digits 9,8,1,1 (or) 9,2,2,2 (or) 9,4,2,1, (or) 3,381 (or) 3,3,4,2. The sum of the digits will be 19 (or) 15 (or) 16 (or) 15 (or) 12 correspondingly. Answer: (D) 28. (32? + 82 + 6)" — (2+2)" is divisible by (A) (82+4) for all positive integers n (B) (32? +.92-+6) for some positive integer n (C) (+2) for all positive integers n. (D) none of these Solution: We know that (a" ~ &*) is divisible by (a — 6) for all natural values of ‘7’, ©, (82? +82 +6)" ~(2+2)" is divisible by 32?4+82+6—2—2 =3n? +7244 = (e+ 1/8244). Answer: (A) 29. Out of the 720 six digit numbers that can bie formed by using all the six digits 2,3,4, 5,7 and 8 the number of such numbers divisible by 11 is (Ayo (B) 36 (©) 72 () 96 Solution: Let the sum of the odd placed digits be and the sum of the even placed digits be y. Now 2+y = 29; zy = —11,0 or 11 since the 6-digit numbers are divisible by 11. Problems and Solutions 91 Since 5 +y =29, either x is.odd and y is even or is even and y is odd. Hence x—y #0, Thus from s—y=—l1 and z+y=29 we get =9, y z+y=29, we get r=20, y=9. The number of such possibilities for 2,y from the corresponding sets of number elenients {8,7,5} or {2,3,4} is 3! each. <. Number of such 6-digit numbers =2 x 3! x 3! = Answer: (C) 30. A triangle has circum-radius R and sides a,b,c with R(b+c)=a-Vbc Then the triangle is (A) acute (B) obtuse (C) right (D) scalene but not right Solution: Given R-(b+¢)=a-Vbe (or) Q) But, by A.M-G. (2) From (1) and (2), we have bee 2 2 R(b+ 0) = avbe = a Gte) :. @=2R = diameter of the circum circle. Hence ZA= 90°. Answer: (C) a RAMANUJAN CONTEST FINAL TEST - 2007 1, Two circles of radii R and r(R > r) touch each other internally at A. ABC is an equilsteral triangle for which B is on one circle and C is on the other. Show that the length of the side of the triangle is Br Ve—Rr+e Solution: Let AP, AQ be the diameters of the circles, Let side of equilateral triangle be < units. + AB = BO = AC == units. Let AC meet the inner circle at D. Now £PDA = ZQCA = 90° (Angles in the semi circle are 90°) => PD||Qc. g D B ey \y/ A fence} AADP ~ [eae] oraz AD _ or _ ar AG = 4Q 7 > =TR™R or AD= 5. (1) Consider ABDA. By sine formula, BD ano = 27> BD=Vier (2) 92 Problems and Solutions 93 Also by cosine formula, BD? = AD?+AB?—2.AD.AB.cos 60° 2 f2r\F og gar) (V3 = (F) 422-2525 > 3 = > oP = => BR? = => side of equilateral A = Hence proved. y Given triangle ABC where AB = AG and ZA = 80°, 8 point M is taken inside the triangle such that ZMBC = 30° and ZMCB = 10°. If the angle bisector of BAC meets BM extended in NV’, then 1. prove that ABNC is isosceles. and 2. Find Z.ANC For Solution , see page: 247 s Find the number of subsets A of X = {1,2,3,--- ,9,10} such that A has at least two elements and A does not contain consecutive elements of X. Solution. Let us first give a lemma for a general problem. Let X = -{1,2,3,-++,n}. Let A C X, such that A has r elements, r€ N, 2, a3 — a2 2 2,...,4r—Gr_1 22. ‘Then f(A) = {a;,02—1,--- a —(r—1)}. This is a subset of {1,2,3,---,n-r+l}. Hence f(A) € 2. ‘This mapping is clearly one-to-one. This is onto. For B= {by,b2,b3,-+- ,be} CY, then A = {bbz + 1,b3 +2,-+- ,be-+r—-1} CX and f(A) = B. .. f 48 one-one and onto; Isl=l21=("7) Now, in the given problem set A can have 2,3,4,5 elements and n= 10. Number of sets A with [4] =2 is (9) =36 Number of sets A with |4|=3,4,5 is (8) = 56, (2) =35 and (§) =6 respectively. -*. Total number of sets is 133, 4. A Swiss bank cellar is protected by 7 security men. The bank cellar hus 10 doora and every door has 3 locks. All locks ar¢ different from gach other. Each security person has the keys to some locks so thet any four security persons have keys to all locks, so that they can open tho cellar in an emergency. Prove that there exist three security persons who together have key5 for all locks. Problems and Solutions 95 For Solution , see page: 246 5. Let R* denote the set of all positive real numbers. Find all functions f : R+ — Rt satisfying f(z. f(y)) = f(zy) +2- Solution: S(z- f(y) = (ey) +2 (given) @) for z,y €R*. Interchanging x,y, we get Fy f(@)) = Hus) + (2) Replacing = by f(z) in (1), we get F(F(@)- FW) = Fly F(z) + £2) (3) From (1),(2),(3), we get S(F(@) - Fly) = Flay) + y+ F(a) (4) Interchanging x,y in (4), we get FCF): F(@)) = Fyz) +2 + Fy) (6) From (4) and (5), we get yt fq) =2+ fv) where C’ is a real constant. Substituting f(z) =2+C in (1), we get gefy)+C = sy+C+e = x S(y) Sees > ae(yt+C) = tyte => cytCo = wyte 96 Problems and Solutions =>[C=1 = [F@) =241 |is the required function. at+bt4+3 _ 2b 6. Prove the inequality 7S > Fp for all real numbers a a and b. Solution: Let t= Jat +@+2 where a,bE R= | tz V2. ates ot * Jat +42 ~ 10 P+1_ 21 #710 = tep>24 (Q) Consider V2+ a = 5 = a > (1.5)(1.41) =2.115 as V2> 141 o. | V2-4 Jq > 2.116 > 2.1 Now Now, v2 (@- va+ Se) ¢-va (1-75) 20 . t> V2 and (J <1 as t> V3 1 1 t+p-vV8— W so C442 V2+Jq>2-1 = (1) is proved. Therefore tha required inequality is true. 7. A 10 digit natural number with no repeated digits and leading digit non zero is called a magic number, if it is divisible by 99999. How many such magic numbers are there? Problems and Solutions 97 For Solution , see page: 250 2 Find all pairs of natural numbers 2 and y for which 2? = 4y+3, lem (x,y) where lem (z,y) denotes the least common multiple of the numbers x and y. Solution: Since z is a divisor of 2? and lcm (z,y), 4y is a common multiple of «,y c. Lem. (2,y) =y or 2y or dy. Case I. lem. (x,y) =y = Kez for some KEN Substituting in 2? = 4y+3 lom(z,y), we get 2? =4Ka+3Ke~= Kz (or) y= 7K; Lom. (7K, TK?) = 7K? for any K €N. Gase II. lem. (x,y) = 2y. Thus 2y = Ka where K is odd (Otherwise-K is even => 2 divides y which is already discussed in Case I. The equation becomes x? = 2Kz+3Kz = 5Kxr z = 5K => 2y = 5K?. This implies K is even which contradicts the fact that K is odd. There's no solution in this case. Case ILE. Lem. (2,y) =4y. Thus 4y= Kx where K is odd (with similar arguments as in Case IT) ‘The equation becomes x? = Ka + 3Kz=4Ka 2. 2=4K and y= K? lem. (4K, K*) = 4K? as K is odd. -. The solutions are z 7K, y= TK" forall KEN oz = 4K, y=K? forallodd KEN 98 Problems and Solutions 9. Suppose that (i) 2,y,a@1,@2,-++ is an A.P., (ii) 7, 4,91, 92,"+- is a G.P and (iii) 2,y,h1,ha,-++ is an. H.P., where x,y are non-zero real numbers. If an, gn, fn; aré in G.P., for some n, show that z= ty. Solution. As 2,y are the first 2 terms common to given A.P., G.P. and HLP., we have a it sol fe ti pee On = (nt Tyne, gn = n= ae ay where On,gnfn are n' terms of A.P., G.P., H.P, respectively. pot? _ (n+ Dy - nal -2y (n+ l)e— ny => Pn t Ie ny =o Y(n + Dy — ne] Since g2 = an: hn, we get (or) (n+ Day(y? — 27") = n(yPn*? — g2n42) (1) Assume that y? £22, ‘Then (1) shows that sy >0=> £>0. Let = ¢ such that t>0, t#1. (1) > (n+1)-t- (2-1) =n. ("42 2) > HL tet peep MF) Sy [gmt 4) (2) If t > 0, t #1, we have ( ~1)-("4-r 4) 5 Q for lersn a an 3 0< Seg try = nett 4 1) -yr. ral ‘This contradicts (2) *. y? =a? => Hence proved. r= RAMANUJAN CONTEST SCREENING TEST - 2008 1, The number of terms of the multinomial expression (ay +@2 +.03+ 44)" is (A) 2H): (By timer) (Ey Matt) (py minty (n42) Solution: We know that (a, + a2)" has (m+ 1) terms. (o1-+@2 +03)" = [(a1 +02) -+a3]" has (n+1) terms which are Mag (04 + 2)" + Mey (44 + g)”Mag + Megas + 02)"—7a3 + ++ Trop (O1 +42)" Tah + -++Mey s(x tea) "+03 (a) Now counting the number of terms of each of the term, we get (nt) tnt (n-1te-+241 S1F2434--+¢(nt 1 = (n+1)(n+2) bong 2 [Note the number of terms in each term of the expression (a) is given by (a1 +42) as each of these terms multiplied by a product of a3* x na, will not add to any extra term]. Now for (a; +a2 +43 + a4)” getting nol +4, +a, +n, (4, +4, +4)" +4. stn, (4, $a, Fag YA tot, (G4, +4, ME +n, a and counting the number of terms in each expression we get (n+1)(n+2) , n(n+i) , (n-1)n eee prrtla-1) 3 1x2 aS 100 Problems and Solutions np 1 nth ntl as [> P+ ‘| a kel k=l }[otnies nents) + pevera| v - lln+ 1)(n+2)(an +3) +3(n-+1)(n+2)] o Hlrt Din+ 2)l0n+ 6) 3 x Un + 1)(n+2)(n +3) {a+ Vint 2)in+8) ieee Answer (B), [Noting that the number of terms of the expressions a}, (a; + a2)", (a1 + 92 +08)" and (a; +02+03 +04)” are 1, a+1, GHt2) tiesants) we conjecture that the number of terms of (a; + a2 +03 +--+ +4,)" is Gow Provel] A polygonal prism has 2010 edges. The number of the faces of the prism will be (A) 72 (B) 670 (C) 1005 @) 672 Solution: Let the polygon constructing the prism be of ‘n'sides. Then two polygonal faces will have 2n edges, Then n vertices of corresponding polygons are connected by 2n (verticel) edges to the n vertical faces, Hence, a polygonal prism constructed from a polygon of n sides will have (n+2) faces and 3n edges. “. Gn = 2010 = n = 670 and the number of faces 670 + 2 = 672, Answer (D). Problems and Solutions 101 3. In order to draw a graph of f(x) = az?--b2-4-c, where a,b,c are real numbers, a #0, a table of values was constructed. These values of the function for a set of equally spaced increasing values of x were 3844, 3969, 4096, 4227, 4356, 4489, 4624 and 4761. The one which is incorrect is (A) 4096 (B) 4227 (C) 4761 Solution: Given f(z) = ac? +be+c, a,b,cER, a#0. F(e+h)-f@) = al(e+h)?-27]+0[(2+h)—a]+e-¢ = 2ahz+ah? +bh which is a linear expression in x. Hence (f(e+h)—F (2), (SF (+2h)—F(e+h)), (f(e+3h)—f(2+2h)), --- should form an Arithmetic Progression. Here is a table of differences f(z) F(c+h) f(e-+ 2h) f(e+3h) f(@+4h) f(a+5h) F(z + 6h) f(e+Th) 3844 3969 4096 4227 4356 4489 4624 4761 125 127 131 129 183 185 137 The difference 131 does not fit in the AP, which is produced by 4227. [The correct number should be 42251}. If 4227 is replaced by 4225, the successive differences will be 129,131.133,135 and 137. Answer (B). 102 Problems and Solutions 4. Given a triangle ABC with medians AE, BF,CD;FH is parallel and equal to AB. BH and HE aredrawn. FE extended meets BH in G. Which one of the following is not necessarily correct? (A) FG = $4B (B) BH =DC (C) HE=HG (D) FG is the median of ABFH. Solution: By mid point theorem FEIAD or DB and FE =}AB. FC\\EH and FC = EH (given) FCHE isa |m™ => CH||FE, CH=FE=134B CHIFE, CH\\EG; and hence G is the mid point of 4 HB > =tow=} i 1 BG = 5CH = TAB FG=5AB4+ 743 i 3 4 FG = ie 54B and FG is the median of AFHB (as G is the mid point of BH). Also CH||EG||DB = CH||DB; CH= FE=}4aB=pp . CHBD isa |" ». BH =D¢C. So, it is not necessary that HE = HG. Answer (C). Problems and Solutions 5. The diagram shows three circles externally tangent to each other and to a semicircle. The shaded area is 120 squmits. The area of the unshaded parts of the semi circle in square units is. (A) 30 (B) so! Solution: Let the radius of the semi circle on AB be 2r units, Let the radii of the circles with centers P and Q be z units and y units respectively. Draw PK © perpendicular to QO. 103 (C) 40 (p) 60 ‘The radius of the circle with centre Q is r units. Now, PQ = (r+2) units, OK'= 2 units, QK = (7 — 2) units and OP = OL— PL = (2r~2) units. From right angled APQK , PQ?-QK? = PK? = OP?-OK? ive,, (r+)? -(r-2) = = ar? = ara or 4rz or 8rz = Hence ir or z and similarly y = § units. PK? = (2r-2)?-a? 4r? Qe t 3 units 104 co Problems and Solutions = 2 The shaded area = axnx(5) +a? Sar? pee = 120 sq, units.(given). ‘ar? = 80cm, Area of the semicircle = } x m x dr? = 2nr? = 160cm?. So the area of the unshaded portion is 160 — 120 = 40em?. Answer (C). What could be the possible value of integer ‘a’ if | (10+ + a.7") is divisible by 51 exactly for any natural number 7? (A) 69 (B) 18 (©) 86 (D) 205 Solution: We know that (a+5)|(a™+0*) for odd natural number m. 2 (10-4 7)|(10749 + 72°#2) = 17/(10% 4 4 7n4ty, Since, 10947 + a.72nt1 = joint 4 72m41 4 (g _ 3),724 and 17/20"*? +.4.7"42, 17\(a — 1). Also A= 10+} —7+14(144)7"+1 and we are given 3|A. As 10— 7/104? — 79+, 3 must divide 1+a. Amongst the values of a given only 86 satisfies both the conditions 17|a — 1 and 3ja+1. ; Answer (C). [Note that you can also derive a = 35(mod351) J Let f(n) be the integer closest to Yn. The value of 1 7 1 : 1 Tey + Fey * 755 °° + Tea007 + sabORy 2 Problems and Solutions 105 {Note: f(6) = 2, f(7)=3; f(12)=3; f(43) =4, ete] (4) 88% = (B) 908» (C) a7$_—(D) 895 Solution: We observe that f(n) =m if and only if 1 1 a 1 m5 (m~ 5) cnc (+5) =m -menem +m. ‘Thus the set {n: f(n) =m} consists of exactly (m? +m) — (m? —m) = 2m numbers. ‘The largest n such that f(n) = 44 (44? = 1936 < 2008 < 45? = 2025) is 447444 = 1980. So for m= 1981, 1982,--- ,2008, f(n) = 45. ‘Thus there are 2x1 numbers with f(n) = 1, 2x2=4 numbers with f(n) = 2,---, 2% 44 = 88 numbers with f(n) = 44. Hence i 1 L 1 1 1 La = AKT HAR GHOX GH + BBX = 88 (Note that 2+4+-++88 = 2(1+2+--++44) = 2x 5x Md’ 45 = 1080) Also from n= 1981, 1982,--- ,2008 (28 numbers) H{n) = 45 80 that be: Fiay = 28% = B- ‘Thus the given sum is 88 + 3% = 8835 - Answer (A). ABC is a triangle with side lengths 13, 14 and 16 units. If T is the incentre and FR is its circum radius, then the value of ALBLS! ig equal to (in Sq. units) (A) 130 (B) 64 (©) 128 (B) 100 106 Problems and Solutions Solution: BC = a = 13units, CA = b = Idunits and AB = c= 15units imply atb+e 139414415 Be eget ieee = 21 units Let K,L,M be the feet of the perpendiculars from incentre I on AB,BC,CA respectively. Then AK = AM = $~>a=8 units, BL= BK =s-b=M-U=7 units and CM = CL = s— c= 21-15 =6 units. Area of AABC = yfs(s—a)(s—H)(s—o) = V2x8x 7x6 = 84 sq units. aM BXUxI 65 “Gh” axe ~@ Units. Also A=rs.> KI=IL=IM=$=#=4 units AP.x BI? x CP 8? + 49)(7? +. 42)(62 + 2 APR BEXCE = SLOP OO +H) R 65° = 64x 64 AL x BI x CI > SAE Cr = 64 sq.units, Answer (B). For Aliter , see page: 248 a 2 ‘The degree of the polynomial P(x) which sutisfies the condition 2P(c—1) = (2~15) P(e) is (A)7 — (B)1_— (C)15_—(D) cari not be determined Solution: The given polynomial equation is @P(z—1) = (x —18)- P(a) Q) Problems and Solutions 107 | P(z) (from (1)) = (@ - 1)| P(e 1) > (7 ~ 1) IP) (from (1). "Again (e-)|P@) = (e—2)|P(z-1) = (2- 2)| Plz) = (e-8)|P(z-1) => (x~ 3) |P(z). Continuing this inductively, we arrive at (2 — 14)|P(2). Thus P(z) = x(x — 1)(x~ 2)-++(«—14)q(z) and P(z—1) = (w@—1)(z~2)-+- (@— 18) «g(t - 1) Substituting these two relations in (1) we get x(z - 1)(z—2)-+-(— 15) -q(x—1) = 2(a — 1)(e— 2)---(w ~ 14)(z — 15) - (2) => g(z-1) (wt) ie. g(x) is a constant. Hence P(s) = (2 — 1)(z — 2) +++ (a — 14). ‘Thus the degree of P(x) is 15. Answer (C). 10. Given f : Z—+Zsuch that f(0) 4 (1); f(m+n)+f(mn—1) = {(m).f(n) +2 for all integers m,n. Then the value of (5) will be (A) 53 (B) 26 (©) 191 (D) 69 Solution: Given f : Z + Z such that i(m+n)+ fmn-1)= f(m)- f(n) +2 Q) Form=0, n=0, f(0)+f(-1)=f(0)-f) +2. (2) Form=-1, n=0,f(-1)+f(-1) = f(-¥)- #0) +2. (8) (2)-(3) gives (f(0) — f(—1)) = £(0)(F(0) ~ F(-1)) “ £Q) =1,since f(—1) # f(). +. From (2), 1+ f(-1) = 1? + 2or f(-l) =2. Putting m=-1, n=l in (1), we get f(—2) + #(0) = f(-1)? +2. 108 Problems and Solutions 2. $(-2)=442-155 Putting m= 1,2 = —1 in (1), we get £(0) + f(-2) = f(1)- F(-1) +2 or £(2) = 2. Putting m= 1,n =1 in (1), we get £(2) + (0) = f(1)- FQ) +2 or £(2) = 5. Putting m= 2,n=1 in (1), we get #(3) + (1) = F(2)- f(1) +2 or £(3) = 10. Finally, putting m = 3,n = 2 in (1), we get 2f(5) = f(3)- #(2) +2 or £(5) = 26. Answer (B). LA billiard ball is hit from a point on 1 co the vertical bosrd -of the table close to the | CoH pocket D as shown in the figure. It meets the horizontal board at an angle of 45°, and continues following the | cho arrow. Into which pocket will the ball fall? (A) 4 Bc Solution: The billiard ball starts from 3 cells away from D. On side “AD, hits at 3 cell away from D on CD, hits side BC 3 cells away from C and hits AB at a point 1 cell away from A. Problems and Solutions 109 From this point it will hit 1 cell away from A onside AD then 1 cell away from C on side BC one cell away from C on side CD and it comes to 3 cells away from A on side AD. From this point (follow the arrows shown) it will move as before and in a few steps to the starting point and it will trace the same path without reaching any pocket. ‘Answer (D). 12, Given that 4*+14.4" = 3”+4_3y, where « and y are integers, the value of « — y is (A) 2 (B) 3 (C) -2 (D) -3 Solution: wage a gv gy = 4441) = 3”(34-1) > 2x5 = 3x80 > Mxd = Hx 5x24 > Pek ae W Hence 22-4 =0=y or =2, y=0 So z—-y=2-0=2 Answer (A). 13. Given an equilateral triangle of side 1, squares are drawn on all the three sides. If a rubber band is stretched around the squares, the area of the region enclosed by the rubber band is (A) 342V3 (B) 34 VB (C) 243V3—(D) 4 Solution:’ The rubber band will enclose the area within the hexagon shown in bold line segments, This includes the four triangles and the three squares, 110 Problems and Solutions ‘The area of each of the outer triangle is 3 7 xia xsin320= 2 oq units wie and the area of the interior equilateral A is also 3 sq units and area of each of the squares is 1 sq unit. So the area enclosed by the rubber band is v3 axT +3=3+4 V3 sq, units. Answer (B). 14, The first digit from the left of a 5 digit number is equal to the number of zeros in the number. The second. digit is equal to the number of digits 1, the third digit is equal to the number of digits 2, the fourth digit is equal to the number of digits 3, and the Sfth digit is equal to the number of digits 4. How many numbers have this property? (A) 0 (B)1 (2 (D)3 Solution: Let the 5 digit number (if it exists) be Ap Ai AnAgAa where A; is the number i's among the digits Ao, A1, Ag, A end 44. Ao cannot be 3 or more. For if Ag is 3, then Ay is atleast 1 which means either A: or Ag is nonzero; hence we do not get 3 zeros in the number, Ag being the leading digit cannot be 0 which means Ag is either 1 or 2 (Ay cannot be zero! Why?) A; cannot be 1, for if A, is 1 there are Ag Ar Az Ay Ag tfe tr] ] atleast 2 ones-a contradiction. If Ay = 2, Ag is atleast 1. If Ag is'l, then Ag and ‘Ay both must be zero. But Ap = 1. Hence this is not possible. Problems and Solutions 111 Ag Ai Ag As Ag 1[3 I ] But this means that there are 3 ones in the 5 digit number which means either Aj or Ag is 1. Then there must be a 2 or 4 which is not possible, .°, Theré is no number with Ay = 1 Ay Ar An Ag Ag (2Ti[2To[o Ao = 2 implies A, is atleast 1; so we have 2 zeros one 2 and atleast one 1. Ag = Ag=0 gives the number 2120-0 where we have two 2's, one 1 and 2 zeros, no 3, and no 4. .”. There is only one number of the required kind. Answer (B). If Ar =8, A321. 15. An island is inhabited by both liars and knights. Every knight always tells the truth and each liar always lies. Once an islander A, when asked about himself and another islander B , claimed that atleast one of A and B isa liar. Which of the following statements is true? (A) both are liars (B) both are knights (C) A isa liar, B is a knight (D) B isliar and A isa knight Solution: The choice (A) is not possible, for in this case, the statement becomes true and both ulter a true statement. Again the choice (B) is not possible as the statement becomes a false statement. A cannot be a liar as the statement made by him became true 80 (C) also is not possible Answer is (D) since (A) is a knight and B is a liar and the Statement also is true, Answer (D). 112 Problems and Solutions igi itten in a spiral like 16, The digits 1,2,3,4,5,6,7,8 and 9 sre written in a spiral lik fashion beginning from the central marked cell. Which digit is written on the cell placed exactly 2008 cells above the marked cell? ferope Tlelo{if2i3f4ts 6[3/4]slo]7/8]6 [ (s{z{7{sjolijole 4iifojij}2{2qi 319 [5)4/131342 2/8[7/6[s]4l3 PEE I i (A)1 (B)3 (c-)8 (D) 9 Solution: The square size containing 2008 cell above the marked cell bas side 2x 2008+1 = 4017. (Compare the Solution of Question No. 2 of Junior level, page 21-22). Total number of cells in the square is (4017). The top right most comer of this is (4017)? (mod 9), which is 0 (mod 9) i.e. 9 in this case as ‘0° is not used and instead of ‘0’ 9 is used. The required cell is 2008 cells to the left of the top cell whose entry is 9. Every gt cell to left of 9 will be 9. Therefore 2007 cell will contain number 9 and so the required cell, (one to the left of 2007 cell) contains the number 8. Answer (C). 17. Two semi-circles are constructed as shown in the figure. The chord PQ of the greater circle touches the p Qo smaller circle and is parallel to the diameter of larger circle. If the length of PQ is 10, then what is the area between the semi circles? (A)50 (BY Gr) BD) 113 Problems and Solutions For Solution , see page: 251 Answer (D). i. 1 oo 18. The sum of 3Vi+iVv3 a BV3+3V5 if TV 545) ul tai ** (A) 18 @) © # ©) & Solution: The given sum becomes 2 1 Laas VRSTs WaT _ Ba _Qn+1)VIn—T— (Qn - 1) VInFT ~ 2 Gai @a 1) Gn T Gn FT) a Sort 1) Van—1 - (2n- 1) VMF1 24 Qn F1)@n= [An +1) ~ Gn] Hos 1 1 = 2 (gest vet) (Why?) ge a a - lea Va" v8 V6 V8 285, yi it) _ if d alam . il 15 = 1,34 2 4 ee = Answer (D). 19, The increasing sequence 1,4,5,16,17,20,21,64, --- include all Powers of 4 and all the numbers that can be written as a sum of distinct powers of 4, Then the 63" element of this sequence is (A) 48 (B) 4 (C) 1365. (D) 341 114 Problems and Solutions Solution: The terms of the given sequence will be 4°, 41,49 +. 4}, 42, 4? 4 49, 42 4 41, 4? +424 49, 43,--- Note that 2"+1—1 terms can be written using 4°, 41, 4?,--. ,4r satisfying the conditions of the problem. As 2"+1—1 < 63 implies 2+? < 64 = 28 or 2° < 25. ‘Hence r= 5. Thus the sixty third element of the sequence is 4°+4?44?+434.44+45 = 144+ 16 +64 + 256 + 1024 = 1365. Answer (C). 20. Start with the integers from 1 to 1078. Replace each of them by the digitel roots to get a sequence of 10798 numbers. Let m be the number of 1’s and n be the number of 2's in the resulting sequence. The mn is digital root of a number is sum of the digits, till it is reduced to a single digit [Example digital root of No. 123456789 is —+ 45 => 9] (Ayo @)L (C)2 (D)3 Solution: A number differs from sum of its digits by a multiple of 9 ie. [the digital root of the number is the remainder when it is divided by 9]. Therefore each number eventually reduces to its remainder upon dividing by 9 and instead of taking zero as remainder for the multiple of 9, we take it as 9 and in writing the natural number from 1 to 1020 we come across the remainders. 1,2,3,4,5,6,7,8,9,1,2,3,-+. and up to 1078 — 1, we get all the digits repeating equal number of times ie. up to 1,2,3,-++10?9 — 1, the number of 2's end the number of 1's are equal and the digital sum of 1008 =1. ‘Thus m is 1 more than n; “, m—n=1, Answer (B). 21. A map of @ neighbourhood is shown in the figure. ‘The neighbourbood has the form of a right Zd triangle. All streets are shown in the map. They divide the neighbourhood into blocks that are also right As. Amuthe wants to drive from Problems and Solutions 115 the top end point to the rightmost end point of the neighbourhood. ‘Traffic regulations allow dnly going down (vertically), right (horizontally) or down by a hypotenuse of a block (slant side) How many different routes can Amutha choose from?, (A) 16 (B) 27 (C) 64 () 90 Solution: Let the bottom right point of the configuration be (0,0). ‘The y co-ordinates increase upwards and z co- ordinates increase to the left. ‘The top most point is (4,4). The number of guch paths can be obtained by the following method. Let Tyg) be the umber of required paths from node (i,j) O 6P = 34C > 36P? = 9[36+(3+ Py => 27P-54P—405 = 0 => P?-2P-15 = 0 > P = bem Problems and Solutions 117 AC = V36+64 = 10cm ‘The perimeter of the triangle is AB+BC+OA = 3¥5+11+10em = 243v5em. Answer (A). 23, Inscribed in a right angled triangle is a square having @ common right angle with the triangle, The legs of the triangle are a,b. The perimeter of the square is A) & 8) © & ©) & Solution: AAPQ is similar to AQRB. b-z _ 6 “ "e * ace (4 orab—(a+b)z+a? = 27 P or (a+b)z = ab x — ee we Gry As the side of the square is 2; , the perimeter is 404. Answer (C). 24. P ig taken in the line-segment AB where A is (2,0), B is (0,1) Q is taken in the line segment CD where C is (6,0) and D is (0,6), The shortest length PQ is (A) 2v2 (Bs (G)2 (D) v2 Solution: The equation of line AB is 2+ 2y = 2 and that of line GD is c+y=6, = that they meet (intersect) at the 4th quadrant so the distance between the lines AB and line CD is0. 118 Problems and Solutions To get the minimum distance between the P snd Q where P and Q are points on the line segments. AB,CD it is required totake P at B(2,0) and the foot of 1 r from P to segment ‘CD is Q and length PQ will be minimum. The minimum distance is nt aims _ w 4 E ae Aliter: (Observe the diagram) Equation of AB is x+2y=2 Equation of CD is z+y=6. If P€ AB then the foot of 1 Q from P to GD lies in the segment CD (Check). If (z1,y1) are the co-ordinates of P then 0 <2, <2 and O 9n—k. ©. For any m€ S, s(m) 9n—k. Also, for any m € S, we have only 3 possibilities. on sim) < 92, s(m) =, s(m) > If n is odd, «(m) = % is not possible. If n is odd and if k= % then me & iff m'e S—S, fs a(m) < 2 iff a(m’) > Bo) 2.9 = 25, when k= %, If nis even, clearly no such k is possible. 4. There are exactly 100 lattice points on the circumference of a circle with centre origin. Show that the radius r of the circle will be either an integer or V2 times an integer. : Solution: 2?+y? = r? is the equation to the circle with centre at (0,0) and radius r. The four points (0,-tr) and (+r,0) le on the circle. If r ig an integer, there are 4 lattice points on the circle, Tf r is not an integer, then these 4 points are not lattice points. 128 Problems and Solutions Further, if (a, 9) lies on the circle, then (—a, 6), (a, 8) and (-c, 6) also lie on the circle. (ie.) one point in each of the 4 quadrants. ‘This means in every quadrant on the circle we have 25 points. Further for all (4,8) lying on.the circle (,a) which is the reflection of (a, 8) in the line y= also lies on the circle. ~, ‘There are 12 points on the circle in the first quadrant below y= and 12 points above y=z-and 1 exactly on y ==. ‘This point sey (cz'c) satisfies 27+ 4?.= 1? 2. Wt =r? > r= 2a > V2 times an integer. 5. Find all natural numbers ‘n’ such that n? — 6n — Fi) = 2008 where P(n) stands for the product of the digits of 'n’ represented in the decimal notation. Solution: Let us prove first that P(n) 8032) 49, we have n(4n — 25) = 49 x 171 = 8379 > 8033: when n= 48, we have n(4n —~25) = 48 x 169 = 8016 < 8037" Problems and Solutions 129 . (1) is satisfied by natural numbers ns48 (2) As P(n) > 0, n?—6n— 2008 >0 ie, n(n—6) > 2008 (3) When n= 46, we-have n(n —6) = 46 x 40 = 1840 < 2008; when n=47, we have n(n—6) =47 x 41 = 1927 < 2008; and whenn = 48, we have n(n—6) = 48 x 42 = 2016 > 2008 © (8) is satisfied by natural numbers n> 48 (4) +. From (2) and (4), we see that the only natural number ‘n’ which could satisfy n? — 6n'+ PS) = 2008 is 48. Verification. When n= 48, on 20) = 48? —6 x 48 - 28 = 2304 — 288 ~ 8= 2008. 6. Find ell functions f : 2x Z— R satisfying the following two conditions: @) f(y), f(z) f(2.2) =1 for all 2,y,2€ 2. (i) fle+1,2) =2 for all ze Z~ Solution:Given f(z,y)- f(y, 2): f(%2) =1 for all 2,y,z€ Z. 80, when z=y =z. fla,2)- F(%,2)f(z2) = 1 130 Problems and Solutions or [f(e,2))> =1or f(u,) =1 ay For 2 € Z and y= 2, we have f(z,y)f(ysy) f(y,z) =1 or f(x,y) f(y.2) <1 for any a,y € Z (since f(y,y)=1) ‘Then f(x,2) = f(x,y) f(ys2) = FER for all 2,y,2€ Z i ai , 2.25 fet.) = ie a ten v) (em), Inductively, fev) _ Lz.) Hew) - teh) =40) ig In particular, flew _ 109) flay) = ayeZ, 8 constant, 2-9 for any . f(Gy) = 7. Let ABCD be a convex quadrilateral with AB = BO = CD and AC # BD. Let E be the intersection of its diagonals. Prove that AE = DE if and only if ZBAD+ZCDA = 120°. Solution: Let LBAD= AC = BD, a contradiction! 2a + B+ 28+ a = 180° or a+ 6 = 60° Now, BAD + ZADC =atZEAD+A+ZEDA = atf+ZABB = (a+f)+(atB) = %(atB) = 120°. Thus AE = DB = ZBAD + ZADC = 120°. Conversely, assume that ZADC + BAD = 120°. But ZADC = ZADE+8 and ZBAD = ZEAD+a « ZADE+ LEAD +048 = ZABB+a+4 = 2(a+8) = 120° + a+ = 60° or ZAEB = ZDEC = 60° «, ZDCE = 120° — 6 and ZABE = 120° - a. By law of sines applied to AAEB and ADEC, we get AP DO ___iDE ain(A20" — @) Go ~ sin60® sin(120° = B) » AE =DE, since 120° —0+120°- 8 = 240°- (+f) = ‘240° — 60° = 180°. implies sin(120°- 4) = sin(120° — 8) ‘Also 120° — cy, 120° — (i lie between 0° and 180° and a 8 Otherwise AC-= BD, a contradiction! 132 Problems and Solutions 8 Two circles Ty and I intersect in two points P and Q. The common tangent of T, and Tz, nearer P than Q, touches T; and [2 at A and B respectively. The tangent to [, at P intersects [3 at E (distinct from P) and the-tangent to Ty at P meets Ty at F (distinct from P). Let H and K be two points on the rays AF and BE respectively such that AH = AP, BK = BP. Prove that the five points A,H,Q,K,B lie on the same circle. Solution:Let FP and EP mest AB in X,Y respectively. YA=YP (Tangents from an external point to i XB=XP © circles) congue Let 4FPE=6 ¢2FAP = @ ZPBE = 0 (Using alternate segment theorem.) Let ZYAP =a, ZXBP =. . LYPA = a(AYAP is isosceles) *. 2XPB = 6(AXBP is isosceles) . 2XYP = 2% Bey ele tas eee Problems and Solutions 133 In triangle XYP, 2a+ 28 +0 = 180° @) AP = AH (by construction) ZAHP = 90— 5 (since FHP = 0) = o+B (using *) (=) LAQP = £PAX=0 1 stemate 7 Z2BQP = ZPBX =f (using a) -, ZAQB =a+6=ZAHB (by ++) Hence A,B,Q,H ere concyclic. Similarly A,B,K,Q are also coneyclic, Therefore A, B,H,K,Q are concyclic. RAMANUJAN CONTEST SCREENING TEST - 2009 1. Four children were playing cricket and one of them broke a windowpane. They made the following statements about the event and only one of them lied. Abbas __: It was not me. Bharat: Tt was not me. Chandra: It was Dinesh Dinesh _: It was Bharat The child who broke the window pane is (A) Abbas (B) Bharat (C) Chandra (D) Dinesh Solution: As Chandra and Dinesh make 2 different statements, both cannot be true. One of them definitely lies. Since only one of them lied, the other two must be telling the truth. -. Dinesh was lying and Chandra speaks the truth. ~. Dinesh is the culprit, Answer (D). 2. The number of squares on a coordinate plane with one vertex at A(—2,2) and atleast one of the coordinate axes as axis of symmetry of the square is (a)3 (B) 5 Oe (m7 Solution: There are 5 such squares. Answer (B). 134 Froblems and Solutions 135 3. The number of real solutions of the equation («-+1)(3*—2) = 1 is (A) 0 (B)1 (C)2 (D) more than 2 For Solution , see page: 252 Answer: (C) 4. In a rectangle ABCD,P,Q,R and S are the mid-points of the sides AB, BC,CD and DA respectively and T is the point on RS such that RT = 27S. If the area of ABCD is k times the area “ cl then & is (A) 3 (Cc) 6 (D)8 Solution: oe isa 4 if parallelogram as PQ || AC || RS and PQ = 5Ac = RS. 2 ©. area, of eae = ; area of PQRS. c area of ABCD = F area of ABCD k=4 Answer (B). 5. On a circle of radius 5m, the points A,B,C are chosen such that AB = 5cm and AB is perpendicular to BC. Then ZACB = (A) 75° (B) 60° (©) 45° (D) 30° Solution: radius = 5 cm Let O be the centre of the circle. OA=AB=OB=5 ZAOB = 60°, ZACB _ ZAOB Teg, Answer (D). = 30° 136 Problems and Solutions 6. A triangle is divided into 4 pieces and the area of the 4 regions are given inside the corresponding regions. The value of z is (A) 22 (B) 18 (C) 15.5 (D) 12.5 Solution: Join AP. Let areaof AADP=a 9 B and > areaof AAPE=b 2 a+b =f ¢ eee a ee) Seep eg b _ z+8 ,_ 248 5 15 Hs atb = : 4x +20 +30 +24 ie ¢ = —— = 9x = Tr+44 Qe = 44 z= 2 For Aliter , see page: 251 Answer (A). 7. p(2) = x? + bx +c, where b and c are integers. Ifp(x) is @ factor of both 2‘ +62? +25 and 324+ 4? +282 +5. ‘Then p(1) is (A)1 (B) 2 (C)3 (D)4 Problems and Solutions 137 Solution: 2? + br + ¢ divides both 2 + 62? +25 and 3a + de? + 280 +5 1.22 + br +e divides 3(x*+62?+25)—324—42?_282—5 = 142?—282+70 = 14(2?—20-+5) Since both are quadratic, 2? + br+c=a?-2r+5 pl) =1-245=4 ‘Answer (D). 8.The base of a triangle is equal to 4. The median drawn to the base equals (V6 — V2). One of the base angles is 15°. The acute angle between the median and the base is (A) 30° (B) 45° (C) 60° (D) 75° Solution: M midpoint of 4 BC. BO = 4, AM = V¥2(V3 — 1). One of the basé angles say ZC = 15°. Let MAC = 6 AM _ MC % sinl5 —sin® 2 _ vaV3-1) _y snd = 3-1 22 a sind = 2 6=30° «ZAMB = 0+15° =45° Answer (B). 9. 13.5.7. 99 ies in the interval The value of 5 168 Too Hes = e inter Lt ok © (Toa) ® (aon) (D) none of these 138 Problems and Solutions Solution: Let X = (1) Bees a aek for n>0 ak i< wins ax< i ©) AlsoX > = aw for n>0 ax? =X (3) + “15 Answer (A). 10. A semicircle is inscribed in a right triangle so that its diameter Hes on the hypotenuse and the centre divides the hy into two segments of 15cm and 20cm, potenuse The radius of the semicircle is (in em) (A) 12 (B) 13 (C8 (D) 9 Solution: 0 is the centre of the circle lying on the hypotenuse. D and E are points of contact of the semi circle with sides BC Problems and Solutions 139 and AB respectively AAOE ~ AOCD 4 Answer (A). 11.An acute angled triangle has sides 10, 24 and x. The number of integral values that a can have is (A)4 (B)5 (C)6 (D)7 Solution: Assume a 2, it is an acute angled A. If a?+b? 24. 15 Sa and < 34. 1.15 24? (ie) 2? > 576 — 100 = 476 T= 22,23, 24. When z = 25, 25 is the longest side. Also 10 +24? > 625. 140 Problems and Solution, ©. Tt is an acute angled A. When x= 26, 10? +24? = 267 = 676 ©. Itis aright A. For ¢ = 27,28, angled A. +33 it will form an obtuse . only when z = 22,23, 24,25, we get an acute angled A. Answer (A). 12.The number of triples of consecutive integers (a, b,c) such that P+ P+e iediaees ah Tie is an integer is (A)1 (B) 2 (c)o (D) infinitely many Solution: Let b=n a=n-—1 and c=n+1 @+O+R _(n- 18 +n84(n41)* _ 3n(n? +2) abe In en) n-143) =3("54s ) 54 RES atete is an integer if 32. is an integer e.,) n?—1 divides 9 (ie..) n?~1=1,3,9 or -1,-3,-9 nm -1=1 n=y2not an integer n?—1=—1 n2 n?-1=3 n=+2 n—-1=9 n=+vi0 -1 n=0 = £2 Thus the triples are n?—-1=9 n’ not acceptable (a,b,c) = (1,2,3), or (-3,-2,-1) only, Answer (B)- 13. Three non-collinear points are marked in a plane, The probability of getting an obtuse angled triangle by joining them is (A) 2 ye ie @ Problems and Solutions ae golution: The way this problem can be approached is by taking 3 points on a circle. This is because 3 non-collinear points have a circum circle. Fix a point A on the circle. Let A! be the point diametrically opposite to A on the circle. Then ABC’ will form an obtuse angled triangle iff both B and C fall in the same circle. Hint: You may choose a chord AB and the point C on the minor segment in two ways in the circle formed by A,B,C, arc. (Angle in a minor segment is obtuse.) Probability that AABC will be obtuse = Probability that both B and C are on semicircle I + Probability that both I B and C are on semicircle II radar 2°12°3 2 * Answer (A). 14.A circle touches two adjacent sides of a square and divides each of the other two sides into two segments of lengths 2cm and 23cm. The radius of the circle is (in cm) (A) 15 (B) 16 (©) 17 (D) 18 Solution: Centre of circle O. M,N points of contact of circle with sides AB,AD. OM L AD,ON 1 AB. NO extended meets CD in R. P is the point of intersection of circle with CD. D DP= po=23 DC=% If r is the radius of the circle, then OM = ON =r and OR=25-+, PR=DR-DP=OM-DP="-2. 142 Problems and Solutions -. In AORP OP? =OR? + PR? (ic.,) r? = (25-1)? + (r — 2) r—54r+629 =0 (r—37)(r-17}=0 r=370r17 r =87 not acceptable .r =17 Answer (C). 15.The tangent at P of a circle meets a diameter AB at T. B is nearer to T than A. Then ZATP +2Z/BPT is (A) 90° (B) 120° (C) 150° (D) not a constant angle Solution: If ZATP = 8, then ZPOT = ZPOB =90°-6 «. ZBAP = poled 2 «. ZBPT = me (Angle between tangent and chord equals angle in alternate segment). .. ZATP + 2ZBPT =6 +22") = 90° ‘Answer (A). 16. If the 2009 digit number 505050 --- 50505 is multiplied by 2009, the sum of the digits of the product is Solution: Product of 5050-:-505 (2009 digits ) and 2009 is 1014646464. . 6464545 having 2013 digits. There are 1003 blocks of 64. “Sum of the digits = 1003 x 10+1+04+14+4+4+5+4445 = 10050. Problems and Solutions 143 17. The number of numbers from 1 to 10! having the sum of their digits equal to 3 is Solution: Consider 100 cells corresponding to 100 digits. Lesser digit numbers will have leading zeroes. For example 3is a 1-digit number represented as 00---003 with 99 leading zeroes. Sum of the digits 3, occurs as a single 3, or a2 and a 1 or there 1’s. A Single 3: There are 100 numbers 2 and 1: There are 100P, = 100 x 99 = 9900 numbers Three 1’s: There are 100C3 = memes = 161700 numbers Total = 100+9900+161700 =171700 18. The lengths of the sides AB, BC,CA ofa triangle ABC are 20, 48 and 52 respectively. The distances to these three sides from an interior point D of AABC are x,y,z respectively. Then 52+ 12y +132 equals Solution: Area of the 4 triangle = VO0X 40 x 12x 8 = 480 ‘ = Also a note that it is a right angled triangle with hypotenuse 52. B A 4 c Area = 5 x20. 48 = 480 -, 20r + 48y+52z =2 area of ABC =960 180+ 12y 4132 = 280 = 240 1 144 Problems and Solutions 19.In a sequence each term is defined as the sum of all the preceding terms from the third term onwards. If the 15th term of the sequence is 8192 where the first term is 2, then the second term is Solution: The sequence is of the form a,b,a+b, 2(a+b),27a+ 27b, 23a + 23b,--» where a=2. The n‘ term is 2"-3(a +). 15" term is 2!2(a +6) = 4096(2 + b) = 8192 Answer b=0. 20.15, 17, 25 are the lengths (not necessarily in order) of two sides of a triangle and the altitude to the third side. The area of the triangle is Solution: Altitude to the 3 side of a triangle will be smaller than the sides enclosing the altitude. , 15 is the altitude. AD = 15>, BD = 289-225 = 8 CD = V6%5 — 225 = 20 4 BC = 28. Area of the a d=) x 28x15 =210 Answer (210). 8 D c 1 Z 1 21.1f a+b+e=land a’bte° whereas a,b,c are non zero, then (a + b)ab+ (b+ c)be + (¢ + a)ca = Problems and Solutions 145 Solution: 11a atb+e = 1 -~+7+4-=3wherea,b,c are #0. atbrte i Orn baer Adding 24% , b+e eta c c ct at 1 gatnlylilis a atbte bree 2 @ a oe pa eed =F - Wt dab + (b+ e)be+ (e+ a)ca= 0. Answer 0. 22. k is the largest positive integer such that (2009)* divides 2010! exactly. Then k is Solution: (2009) = 7? x 41 (2009)* = 7 x 41* (2009)* divides (2010)! implies (41)* divides (2010)! and 7 divides (2010)! Maximum power of 7 dividing (2010)! _ [2010] , [2010] , [2010] /2010 e+ Fe] + Fel + Pee] + = 28744145 = 333 Maximum power of 41 dividing (2010)! is — [2010] [2010 | + [Fe | =49+1=50 + KS50 and 2k < 333 Maximum value of & is 50. 146 Problems and Solutions 23. On a square of side length 4om a triangle is constructed outwardly with one side as base. A square inscribed within the triangle has side lem. Then the area of the triangle outside the smaller square is ———— é Solution: Let h be the height of the AEDC. > c AEPQ|||AECD it is given that QP =1cm A B ee ic, dhoa = A 4 3ho= 4 ee 4 8 3 Shaded orca 4x 3 x gol eae ent 24.The set of all real numbers @ which satisfy the inequality 2 < 16?" is —— Solution: 2 < 16% Taking log to the base 2

° When does equality hold? Let the internal angle bisector of A meet the base BC at N’ and the circumcircle of AABC at M. ‘The tangent at M produced meat AP produced at S. NowZBAM = ZMAC = ZCMT = 4 = ZBCM (Angle in the alternate segment theorem and angles in the same segment theorem.) BO||SMT AD _ AN AD, AD _ AN @ DS ~ NM DP~DS NM ) aoe AC ~ NO AB _ AC _AB+AC _AB+AC _bte (y) BN ~ NO BN+NC BC a AMAC is similar to AMCN{ZMAC = ZMCM and 1s1 152 Problems and Solutions ZAMC = ZNMC } AC _AM _ OM “NC CM NM AN ue AM _y NM NM AM CM cM NM Ac\? . (x0) Ea (ye > +e? og, CF _ (a+)? iiss Similarly 3 AD , BE, OF DP EQ’ FR By AM —GM inequality Iv ate be | ca, ab ateta 2 _AD | BE CF “DP * bot FR 2 6+6-3=9 Equality holds when a = b =c 2.How to construct an equilateral triangle which is inscribed in @ given scalene triangle with one side parallel to one side of the scalene triangle, problems and Solutions ee eo Solution: Construct an equilateral A on BC with vertex A’ opposite to A. Join AA’ and let it meet BC in D. Draw lines through D parallel to A’C and A’B, respectively to meet AC’ in E and AB in F. Since DE || A'C, ZDCA! = ZCDE = 60° Since DF || A'B, Z4FDB = ZDBA'=60° -. ZEDF = 60°. Also AAFD|||AABA’ A _ AF _ AD “FB DA () q AB AD AAED|||AACA' ao 7 DA (2) AE _ AF From (1) and (2) aC- TE EF || BC Since EF || BC and ED || CA’ FED = ZEDC= ZBCA' = 60° Similarly ZEFD = 60° -.4 DEF is equilateral with one vertex each on each of the 3 sides of AABC and EF || BC. . Find all integer solutions to the system of equations given below a°+b°-2 = cc b4+ct-2 = ab-a e+a?-2 = Bd Solution: The RHSs of all the 3 equations is of the form (c*—c) Let us compare y*—2 and z°—a with integer values fay Claim: 3" >n3—n+3 for all n>1. 154 Problems and Solutions Proof is by induction. For n = 1,2 it is clearly true. Assume it to be true for some n > 2. Then 3m41 — (n 4133+ (n+1)-3 > 3(n?—n+3) —(n+1)3+(n4+1)-3 2n3 — 3n? —5n +6 (n-1)(n—2)(2n +3) 0 (n +1 —(n 41) 43. Viiv ©. Our claim is proved. Now for all m > 3,m" > 3" >n3-n+3 om™—2>n8—n+1>n3—n for all m>3 and n>1 (A) sm"—2>n3—n for all m>3 and n>1 Let (a,b,c) be an integer solution of the equations. Clearly no two of a,b,c can be zero. By symmetry, assume that a < b < c if a > 3 then 3—4 and a° —a < -6. This implies the second equation of the system has no solution. [Whether 6 or c is positive or negative, if a <—2, b*4e2 > 2 1 +82 > ~4). If a=—1, then the second equation becomes =1 But a=~1,b= 1 does not satisfy the other equations. If a=0 then 0b a = b = c = 2 is a solution of the system. Thus a=b=c=1, and a= 6 =c = 2 are the only integer solutions to the system. Let Ay, 42, Ag, 44, As, Ao be distinct points in a plane. D and d be the longest and shortest distances respectively between If a = 2,2 < 6 < c. The second equation becomes s pairs of points among them. Show that. 2 >v3 Solution: Let ABC be a triangle with ZB > = am od Then 0? = a? +c —2accosB cos B < cos = —5 IV 1 O+e+ pac a? +c +ac Let a 3a”. wbevi a So in the present situation we need to find a triangle whose one angle is > = i 156 Problems and Solutions If the six points form a convex hexagon, then the six angles of ar the hexagon sum to 4r-; so at least one is > =*. Then clearly 2 > V3. If they do not form a convex hexagon, then one of the points must be in the interior of the convex hull of others. Draw diagonals to triangulate the hull as given below. then the interior point P lies inside or on one of the triangles say ABC. Then one of the angles ¢, ZAPC,ZBPC and oo ZAPB is > 2, 8 4 Hence 2 > v3. 5.Determine the six-digit perfect squares each having the property that if each digit is increased by one, the resulting number is also a perfect square. Solution: Let the 6 digit square be m = 2-10? +b-108 +6. 109 +.d-10? 40-104 F Assume n? = (a+ 110° + (6+ 1)108 + (e+ 1)10° +(4+ 11? + (e+ 1104 f 41 111111 d; Now 111111 =3x7x 11x 13x37 and d; can take one of 32 different divisors. Since n > m and positive d;> = M1 ntm=d n—-m= 111111 There are at most 16 solutions given by m=3(4,_ un 2 q Problems and Solutions 157 Since m? is a 6 digit number 1 < 100VI0 < m<1000 200V10 < The values that d; can take are TIITIT d =| m| mm & 3-13-37 = 1443 77 | 683 | 466489 | 577600 | 760 3-7-37= 777 143 | 317 | 100489 | 211600 | 460 3-11-37 = 1221 91 | 565 | 319225 | 430336 | 656 7-11-37 = 1001 111 | 445 | 198025 | 309136 | 556 ne n 6. A hexagon is inscribed in a circle of radius 1. Alternate sides have length 1. Show that the mid points of the other three sides form an equilateral triangle. Solution: Bach side of length 1 forms an equilateral A with the centre. Let the other 3 sides make angle 20,26 and 2y at the centre ZBOC = 2a LDOB = 26 and LFOA=2y. The midpoints of BC,DE,FA are Q,R and P respectively. Then 0Q = cosa, OR = cos, OP = cosy PQ? = OP? +0Q? -20P- OQ cos(7+ a + 60) = cos? a + cos*-y — 2cos a cos y[cos(‘y + @) cos 60 —sin(7 +a) sin 60] VOtB+y=90° inB sin(a +6) = cos. a+y=90-f8 — cos(a+7) = 158 Problems and Solutions Therefore, v3 ‘ PQ?= cos*a + 0s" + 2cosa cosy cos B—- — cosa cosysin g = cosa + cos® 9 + cos” + V3 cos acos 6 cosy — cos aos sin B — cos Bsin(a +7) = cos” a + cos? 8 + cos?-y + V3. cos acos 8 cosy —cos a sin i cosy — sin a.cos 3 cosy — cos cos A siny ‘This expression is symmetric in a, and <. APQR is equilateral. Consider the number of positive even divisors for each of the first n positive integers and form the sum of these numbers. Form a similar sum of the numbers of positive odd divisors of the first n positive integers. Prove that the two sums differ by at most n. x Solution: We know that the number of integers divisible by d among 1,2,3,-++ ,n is 5 Let o(k) and e(k) be the number of positive odd and even divisors of k. You) Trivially Yolk) - > e(k) cost = Yolk) - Keay k= ka Problems and Solutions 159 3, For positive real numbers a,b,c with a+b+c=1 prove l+a l-a 1 Solution: a+b+c=1 lta _2atbt+e_,, 2a l-a@ b+e be The inequality is equivalent to 2a 2 2c bic,a ita Lee, (Sret3) + (Grat})+ (Gath) <0 +59 To prove that a _be , _a 3 c(b+ce) | a(c+a) b(at+b) ~ 2 ‘To prove that ay? Pe ed 3 Geb+e) * abeleta) * abela+s) 2 ia) We use Cauchy Schwartz inequality which states (cin +222 + aus)? < (oP +2} +28) +93 + ¥8) Mee eee een eS ou eaty Vabe(btc) Vabc(e+a) Vabe(a +b) = Vabe(b+Fe) y2= Vabe(e+a) ys= Vabe(a + b) (ae tie? ea! ca re * hele +a) + aioe) (abe{o + ob abe(c +a) + abe(a + 6)] > (ab + be + ca)? 160 Problems and Solutions ab? Be? ca? (ab + be + ca)? (es) Febae) * abolera) * abc(atb) ~ 2abc(a+b+e) So to prove I if is enough to prove that (ab+be+ca)? . 3 Qabe(a + b+c) ~ 2 (i.e.,) Enough to prove that (ab +be + ca)? > 3abe(a + b+) But a+b+c=1 . It is enough to prove that (ab + be + ca)? > 3abe But (c+ yt 2)? > 3(cy + yz + zx) 1 erty +2? —ay—ys—2x = 5((e—y)? + (y— 2)? + (z—2)") 0 Iv o(e@tytzP > Ste + ys +22)| J (ab+be+ca)? > 3[ab-be+ be-ca+ ca ab] = Babe(a+ b+) = 3abe Hence backtracking the required inequality is proved. Y RAMANUJAN CONTEST SCREENING TEST - 2010 ‘The number of polynomial functions f of degree > 1 satisfying § (2?) = (fe)? = (F(@)) is (A) 0 (B)1 (c)2 (D) infinitely many Solution: Let degree of f(z) be n. deg of f(z”) = 2n deg of (f())? = In deg of f(f(z)) = 0° n? = 2n iff n=0 or 2, But n21 Degree of f(z) =2 Since f(a?) = (F(z))®, if the leading coefficient is a then the coefficient of x* in.the LHS is a aiid iti the RHS is a”. a=a? = a=1- {a=0 not admissible) f(z) x? +brte fe). = of pbe® ea (fa) se? thn of = at 4 abn + Bx + 2e0? +o? + Dhex Thus == 2b; 7 b50 “Et Loemb 3 %c=02c=0 . The only function is f(x) = a? Answer ; (B). Two circles touch internally.at A and APQ isa chord which | cuts them in P and Q. The tangent at P meots the other circle at H and K. If HQ = 20llom, then K@ is cqual to (in cm) 162 Problems and Solutions (A) 4011 (B) $(2011) (GC) 1005.5 (D) none.of these Solution: Let AT be the common tangent at-A. Join AK. Let it meet the smaller circle at L. CPAP = ZPLA in-the smaller circle. ZQAT = CTAP = ZQKA in the bigger circle. ZQKA=ZPLA Hence QK || PL. 3 As KP. is transversal to the parallel lines QK and PL _£QKP=0 =. LKPL= PAL langle in alternate segment, as K PH is tangent to inner circle] ZQAK = ZQHK In AQKH, ZQKH = ZQHK. QH = QK = 2011. Answer: (D) 3. The number of real roots of thé equation 14¢+224-23 ib : (A) 0 (B)1 (C)3 (D)t Solution: . Q+a)+e7+2) = of(1+2) (i+a)[.+22~24] = 0 Problems and Solutions 163 Either 2 = —1 Then 2? = If 2 is real x? So 2? z= +(e +. 8 real foots —1, +f et 1 [vot Answer = 4. (10), (34 . (1,4) are the vertices of a quadrilateral. ‘The equation ‘of the line through the origin which divides‘the quadrilateral into two parts of equal area is (A)y=$¢ @)y=§e Oe=ty OM) usar Solution: Area of the trapezium ABCD is $4 x2=9. ¥ as, Area of the trapezium APQD is 5 Equation of OQP is y= ma say Pis(3,3m) Q is (1,m) 164 Problems and Solutions $ i 3 Area of APQD = ">" x2=4m 24m = m= t loool .. eqn to line is y Answer: (A): 5. The number of pairs (a, 5) of positive integers that satisfy the equation ab 24 = 2c is (ayo (B)2 . (c)6 (D) 8 Solution: ab-2a = 24 a(o-2) = 24 (a,2) = (1,28) (a,b) = (2,34) (a,b) = (3,10) (@,) = (4,8) (a,b) = (6,6) (a,b) = (8,8) (0,8) = (12,4) (0,6) = (24,8) Answer : (D) Note: The number of integer factors of 24 = 25 x 3! is 4 2=.8, For each factor of 24 assigned to a, wo-get'an integer solution for b. Hence 8 solutions. problems and Solutions 166 80 1 . 6. The eae Lnrtitesne ie (A) io (8) % ©) wb ©) % Solution: 1 = riveFT+(k+1)VE kV +1 + (b+ IVE (h+ 1) — KCK +1) (k+1)Vk- kVRF1 K(k $1) Sooo ~ VeRO Vert 3 ri : Ff. : ) “Coke T+ (R+iVe ~ \Ve Jeri Answer: (D) Let (fn) be a sequence defined by fy = fa = 1 and fni2 = fra + fa for _n > 1.-Then the area of the triangle with side lengths Vfone1, V/fant2s V/fanss, is (A)1 (B) 4 : (Cy } (D) none of these Solution: Due to the condition given (WV Fancea)® = (Fant)? + (s/ Fanta)? +. The triangle with the given sides is a’right angled triangle Its area is 5V. Fanvifinea- But this goes unbounded as n becomes larger and larger. Honce none of thé 3 choices. given fit the solution. Answer: (D) Note:--It could be modified as thg atea.of the smallest. sch triangle, Then the answer will be a= BVA tf

You might also like